HESI Fundamentals practice

Ace your homework & exams now with Quizwiz!

What client statement indicates to the nurse that the client requires assistance with bathing?

"I don't understand why I'm so weak & tired."

A male nurse is assigned to care for a female Muslim client. When the nurse offers to bathe the client, the client requests that a female nurse perform this task. How should the male nurse respond? "May I ask your daughter to help you with your personal hygiene?" "I will ask one of the female nurses to bathe you." "A staff member on the next shift will help you." "I will keep you draped and hand you the supplies as you need them."

"I will ask one of the female nurses to bathe you."

Male nurse is assigned to care for female Muslim client. When nurse offers to bathe the client, the client requests that a female nurse perform this task. How should the male nurse respond?

"I will ask one of the female nurses to bathe you."

The nurse is instructing a client with high cholesterol about diet and life style modification. What comment from the client indicates that the teaching has been effective? "If I exercise at least two times weekly for one hour, I will lower my cholesterol." "I need to avoid eating proteins, including red meat." "I will limit my intake of beef to 4 ounces per week." "My blood level of low density lipoproteins needs to increase."

"I will limit my intake of beef to 4 ounces per week."

5. Which statement about the use of heat and cold for osteoarthritis pain by the client is of most concern to the nurse?

"I wrap the heating pad around my knee and use an extension cord so I can walk around the house."

At the time of the first dressing change, the client refuses to look at her mastectomy incision. The nurse tells the client that the incision is healing well, but the client refuses to talk about it. Which is the best response to this client's silence? "It is normal to feel angry and depressed, but the sooner you deal with this surgery, the better you will feel." "Looking at your incision can be frightening, but facing this fear is a necessary part of your recovery." "It is OK if you don't want to talk about your surgery. I will be available when you are ready." "I will ask a woman who has had a mastectomy to come by and share her experiences with you."

"It is OK if you don't want to talk about your surgery. I will be available when you are ready."

19. Susan's daughter, who remained at Susan's bedside throughout the night, asks the purpose of this medication (Lovenox). Which response is correct?

"It is a type of anticoagulant that is given to prevent venous blood clots."

A 4y/o boy who is scheduled for a tonsillectomy & adenoidectomy asks the nurse, "Will it hurt to have my tonsils & adenoids taken out?" Which response is best for the nurse to provide?

"It may hurt, but we'll give you medicine to help you feel better."

Daughter of an older woman who became depressed following the death of her husband asks, "My mother was always well-adjusted until my father died. Will she tend to be sick from now on?" Which response is best for the nurse to provide?

"It's highly likely that she will recover & return to her pre-illness state."

The daughter of an older woman who became depressed following the death of her husband asks, "My mother was always well-adjusted until my father died. Will she tend to be sick from now on?" Which response is best for the nurse to provide?

"It's highly likely that she will recover and return to her pre-illness state."

Single mom was just told she has advanced cancer. She expresses concern about who will care for her children. Which statement made by the nurse is likely to be most helpful at this time?

"Tell me what you would like to see happen with your children in the future" Rationale: Nurse should first assess what the client desires.

A single mother of two teenagers, ages 16 and 18, was just told that she has advanced cancer. She is devastated by the news, and expresses her concern about who will care for her children. Which statement by the nurse is likely to be most helpful at this time?

"Tell me what you would like to see happen with your children in the future."

A single mother of two teenagers, ages 16 and 18, was just told that she has advanced cancer. She is devastated by the news, and expresses her concern about who will care for her children. Which statement by the nurse is likely to be most helpful at this time? "Your children are old enough to help you make decisions about their futures." "The social worker can tell you about placement alternatives for your children." "Tell me what you would like to see happen with your children in the future." "You have just received bad news, and you need some time to adjust to it."

"Tell me what you would like to see happen with your children in the future."

When the nurse enters a client's room to do an initial assessment, the client shouts, "Get out of my room! I'm tired of being bothered!" How should the nurse respond?

"What is concerning you this morning?"

When the nurse enters a client's room to do an initial assessment, the client shouts, "Get out of my room! I'm tired of being bothered!" How should the nurse respond? "There is no reason to be so angry." "Why do I need to leave your room?" "What is concerning you this morning?" "Let me call the client advocate for you.

"What is concerning you this morning?" Rationale (C) is an open-ended question that encourages the client to discuss personal feelings. (A) devalues the client and hinders further communication. Acting defensively and asking "why" questions such as (B) are likely to elicit more anger and block communication. By deferring to the client advocate (D), the nurse fails to even address the client's feelings of anger and exasperation.

When the nurse enters client's room, client shouts, "Get out of my room! I'm tired of being bothered!" How should nurse respond?

"What is concerning you this morning?" Rationale: open-ended question that encourages client to discuss personal feelings

A client who is 5 foot 5 inches tall and weighs 200 pounds is scheduled for surgery the next day. Which question is most important for the nurse to include during the preoperative assessment? "What is your daily calorie consumption?" "What vitamin and mineral supplements do you take?" "Do you feel that you are overweight?" "Will a clear liquid diet be okay after surgery?"

"What vitamin and mineral supplements do you take?"

26. Susan asks the nurse why she is receiving two medications to prevent clots (Lovenox & Coumadin). How should the ICU nurse answer Susan's question?

"You will receive enoxaparin (Lovenox) injections and warfarin (Coumadin) pills until the target anticoagulation numbers are reached."

Heparin 20,000 units in 500 ml D5W at 50 ml/hour has been infusing for 5½ hours. How much heparin has the client received? A. 11,000 units. B. 13,000 units. C. 15,000 units. D. 17,000 units.

(A) is the correct calculation: 20,000 units/500 ml = 40 units (the amount of units in one ml of fluid). 40 units/ml x 50 ml/hr = 2,000 units/hour (1,000 units in 1/2 hour). 5.5 x 2,000 = 11,000 (A). OR, multiply 5 x 2,000 and add the 1/2 hour amount of 1,000 to reach the same conclusion = 11,000 units. Correct Answer: A

The healthcare provider prescribes furosemide (Lasix) 15 mg IV stat. On hand is Lasix 20 mg/2 ml. How many milliliters should the nurse administer? A. 1 ml. B. 1.5 ml. C. 1.75 ml. D. 2 ml.

(B) is the correct calculation: Dosage on hand/amount on hand = Dosage desired/x amount. 20 mg : 2 ml = 15 mg : x . 20x = 30. x = 30/20; = 1½ or 1.5 ml. Correct Answer: B

A client is to receive 10 mEq of KCl diluted in 250 ml of normal saline over 4 hours. At what rate should the nurse set the client's intravenous infusion pump? A. 13 ml/hour. B. 63 ml/hour. C. 80 ml/hour. D. 125 ml/hour.

(B) is the correct calculation: To calculate this problem correctly, remember that the dose of KCl is not used in the calculation. 250 ml/4 hours = 63 ml/hour. Correct Answer: B

At the time of the first dressing change, the client refuses to look at her mastectomy incision. The nurse tells the client that the incision is healing well, but the client refuses to talk about it. What would be an appropriate response to this client's silence? A. It is normal to feel angry and depressed, but the sooner you deal with this surgery, the better you will feel. B. Looking at your incision can be frightening, but facing this fear is a necessary part of your recovery. C. It is OK if you don't want to talk about your surgery. I will be available when you are ready. D. I will ask a woman who has had a mastectomy to come by and share her experiences with you.

(C) displays sensitivity and understanding without judging the client. (A) is judgmental in that it is telling the client how she feels and is also insensitive. (B) would give the client a chance to talk, but is also demanding and demeaning. (D) displays a positive action, but, because the nurse's personal support is not offered, this response could be interpreted as dismissing the client and avoiding the problem. Correct Answer: C

The healthcare provider prescribes the diuretic metolazone (Zaroxolyn) 7.5 mg PO. Zaroxolyn is available in 5 mg tablets. How much should the nurse plan to administer? A. ½ tablet. B. 1 tablet. C. 1½ tablets. D. 2 tablets.

(C) is the correct calculation: D/H × Q = 7.5/5 × 1 tablet = 1½ tablets. Correct Answer: C

Twenty minutes after beginning a heat application, the client states that the heating pad no longer feels warm enough. What is the best response by the nurse? A. That means you have derived the maximum benefit, and the heat can be removed. B. Your blood vessels are becoming dilated and removing the heat from the site. C. We will increase the temperature 5 degrees when the pad no longer feels warm. D. The body's receptors adapt over time as they are exposed to heat.

(D) describes thermal adaptation, which occurs 20 to 30 minutes after heat application. (A and B) provide false information. (C) is not based on a knowledge of physiology and is an unsafe action that may harm the client. Correct Answer: D

An IV infusion terbutaline sulfate 5 mg in 500 ml of D5W, is infusing at a rate of 30 mcg/min prescribed for a client in premature labor. How many ml/hr should the nurse set the infusion pump? A. 30 B. 60 C. 120 D. 180

(D) is correct calculation: 180 ml/hr = 500 ml/5 mg × 1mg/1000 mcg × 30 mcg/min × 60 min/hr. Correct Answer: D

The nurse mixes 50 mg of Nipride in 250 ml of D5W and plans to administer the solution at a rate of 5 mcg/kg/min to a client weighing 182 pounds. Using a drip factor of 60 gtt/ml, how many drops per minute should the client receive? A. 31 gtt/min. B. 62 gtt/min. C. 93 gtt/min. D. 124 gtt/min.

(D) is the correct calculation: Convert lbs to kg: 182/2.2 = 82.73 kg. Determine the dosage for this client: 5 mcg × 82.73 = 413.65 mcg/min. Determine how many mcg are contained in 1 ml: 250/50,000 mcg = 200 mcg per ml. The client is to receive 413.65 mcg/min, and there are 200 mcg/ml; so the client is to receive 2.07ml per minute. With a drip factor of 60 gtt/ml, then 60 × 2.07 = 124.28 gtt/min (D) OR, using dimensional analysis: gtt/min = 60 gtt/ml X 250 ml/50 mg X 1 mg/1,000 mcg X 5 mcg/kg/min X 1 kg/2.2 lbs X 182 lbs. Correct Answer: D

The UAPs working on a chronic neuro unit ask the nurse to help them determine the safest way to transfer an elderly client with left-sided weakness from the bed to the chair. What method describes the correct transfer procedure for this client? A. Place the chair at a right angle to the bed on the client's left side before moving. B. Assist the client to a standing position, then place the right hand on the armrest. C. Have the client place the left foot next to the chair and pivot to the left before sitting. D. Move the chair parallel to the right side of the bed, and stand the client on the right foot.

(D) uses the client's stronger side, the right side, for weight-bearing during the transfer, and is the safest approach to take. (A, B, and C) are unsafe methods of transfer and include the use of poor body mechanics by the caregiver. Correct Answer: D

50.The nurse assesses an immobile, elderly male client and determines that his blood pressure is 138/60, his temperature is 95.8° F, and his output is 100 ml of concentrated urine during the last hour. He has wet-sounding lung sounds, and increased respiratory secretions. Based on these assessment findings, what nursing action is most important for the nurse to implement? A. Administer a PRN antihypertensive prescription. B. Provide the client with an additional blanket. C. Encourage additional fluid intake. D. Turn the client q2h.

(D) will help to move and drain respiratory secretions and prevent pneumonia from occurring, so this intervention has the highest priority. Older adults often have an increased BP, and a PRN antihypertensive medication is usually prescribed for a BP over 140 systolic and 90 diastolic (A). Older adults often run a lower temperature, particularly in the morning, and (B) does not have the priority of (D). Even though the client has adequate output, (C) might be encouraged because the urine is concentrated, but this intervention does not have the priority of (D). Correct Answer: D

13. Which are common components verified during an operating room time out? (Select all)

-Client identity -Procedure to be done -Surgical site -Allergies

9. The affected extremity should be assessed for which of the following neurovascular checks are performed? (Select all)

-Color -Temperature -Sensation

16. Which task(s) are required before transfusing this unit of PRBCs? (Select all)

-Confirm the right client, the right blood type, and the right unit of blood with another RN -Ensure that the consent is signed and current -Gather supplies, including new blood tubing, new normal saline to hang with the blood, and a pump if it is required by hospital policy

21. When administering enoxaparin (Lovenox) in a prefilled syringe, which action(s) should the nurse take? (Select all)

-Ensure that the bubble is nearest to the plunger end of the syringe -Administer in the posteriolateral or anteriolateral abdomen

8. Which clinical manifestation(s) of the affected extremity should the nurse practicing in the emergency department expect? (Select all)

-External rotation -Shortening -Muscle spasms

2. The nurse expects to see which manifestations of osteoarthritis in Mrs. Weil? (Select all)

-Joint pain -Swollen nodes of the joints -Asymmetrical involvement of the joints

31. Which resources are appropriate to promote safety and independence for the client? (Select all)

-Shower chair -Medical alert system -Home healthcare referral -Raised toilet seat

7. Which additional information is associated with an increased risk for falls? (Select all)

-Throw rugs placed on hardwood floors in the home -Walks with short, shuffled steps -Uses the furniture to steady self -Advanced age

18. If the current order is cefazolin (Ancef) 2 grams every 6 hours for 24 hours, when will the last dose be given?

0100 on Thursday

Medication is prescribed to be given QID. What schedule should the nurse use to administer this Rx?

0800, 1200, 1600, 2000

A medication is prescribed to be given QID. What schedule should the nurse use to administer this prescription?

0800, 1200, 1600, 2000.

The nurse working in the ED is assessing 4 clients' ability to tolerate pain. Which client is likely to tolerate a higher level of pain?

1 55y/o woman who has had moderate low back pain for 3 months Rationale: Experiences with the same type of pain that has successfully been relieved makes it easier for the client to interpret the pain sensation and, as a result, the client is better prepared to take steps to relieve the pain. All other clients are having new experiences with pain.

Nurse is preparing patient with catheter & IV infusion to ambulate from bed to chair for first time following abdominal surgery. What action(s) should nurse implement prior to assisting client to the chair?

1. Premedicate with an analgesic 2. Inform client of plan for moving to the chair 3. Ask client to push IV pole to the chair 4. Assess the client's BP

A low-sodium, low-protein diet is prescribed for a 45y/o client with renal insufficiency & HTN, who gained 3lbs in the last month. The nurse determines that the client has been noncompliant with the diet, based on which report from the 24hr diet recall?

1. Snack of potato chips & diet soda 2. Lunch of tuna, carrots, fruit & coffee 3. Breakfast of eggs, bacon, toast & coffee 4. Bedtime snack of crackers & milk

A client is receiving alprazolam (Xanax) 0.75 mg PO bid for anxiety. Alprazolam is available in 0.5 mg scored tablets. How many tablets should the nurse administer? (Enter numeric value only.)

1.5

Seconal 0.1 gram PRN at bedtime is prescribed to a client for rest. The scored tablets are labeled grain 1.5 per tablet. How many tablets should the nurse plan to administer? A. 0.5 tablet. B. 1 tablet. C. 1.5 tablets. D. 2 tablets.

15 gr=1 Gm. Converting the prescribed dose of 0.1 grams to grains requires multiplying 0.1 × 15 = 1.5 grains. The tablets come in 1.5 grains, so the nurse should plan to administer 1 tablet (B). Correct Answer: B

20. Enoxaparin (Lovenox) 40mg is prescribed subcutaneously once daily and is supplied in a prefilled syringe containing 20mg/mL. How many mL should a prefilled 40mg syringe contain?

2

A client who has a sinus infection is receiving a prescription for amoxicillin/clavulanate potassium (Augmentin) 500 mg PO q8 hours. The available form is 250 mg amoxicillin/125mg clavulanate tablets. How many tablets should the nurse administer for each dose? (Enter numeric value only.)

2

A client with pericardial effusion has phrenic nerve compression resulting in recurrent hiccups. The healthcare provider prescribes metoclopramide (Reglan) liquid 10 mg PO q 6 hours. Reglan is available as 5 mg/5 ml. A measuring device marked in teaspoons is being used. How many teaspoons should the nurse administer?

2

A client with type 2 diabetes is receiving metformin (Glucophage) 1 gram PO twice daily. The medication is available in 500 mg tablets. How many tablets should the nurse administer? (Enter numeric value only.)

2

30. What is the maximum number of tablets that Susan is allowed to take in 24 hours without exceeding the maximum of 2000 mg

4

Evaluate the client's mental status for competence to refuse treatment.

A 35-year-old female client with cancer refuses to allow the nurse to insert an IV for a scheduled chemotherapy treatment, and states that she is ready to go home to die. What intervention should the nurse initiate?

It may hurt but we'll give you medicine to help you feel better."

A 4-year-old boy who is scheduled for a tonsillectomy and adenoidectomy asks the nurse, "Will it hurt to have my tonsils and adenoids taken out?" Which response is best for the nurse to provide?

The nurse working in the emergency department is assessing four clients' ability to tolerate pain. Which client is likely to tolerate a higher level of pain?

A 55-year-old woman who has had moderate low back pain for three months.

Foods and liquids consumed during the past 24 hours.

A 73-year-old Hispanic client is seen at the community health clinic with a history of protein malnutrition. What information should the nurse obtain first?

31.When making the bed of a client who needs a bed cradle, which action should the nurse include? A. Teach the client to call for help before getting out of bed. B. Keep both the upper and lower side rails in a raised position. C. Keep the bed in the lowest position while changing the sheets. D. Drape the top sheet and covers loosely over the bed cradle.

A bed cradle is used to keep the top bedclothes off the client, so the nurse should drape the top sheet and covers loosely over the cradle (D). A client using a bed cradle may still be able to ambulate independently (A) and does not require raised side rails (B). (C) causes the nurse to use poor body mechanics. Correct Answer: D

A client is receiving a cephalosporin antibiotic IV and complains of pain and irritation at the infusion site. The nurse observes erythema, swelling, and a red streak along the vessel above the IV access site. Which action should the nurse take at this time? A. Administer the medication more rapidly using the same IV site. B. Initiate an alternate site for the IV infusion of the medication. C. Notify the healthcare provider before administering the next dose. D. Give the client a PRN dose of aspirin while the medication infuses.

A cephalosporin antibiotic that is administered IV may cause vessel irritation. Rotating the infusion site minimizes the risk of thrombophlebitis, so an alternate infusion site should be initiated (B) before administering the next dose. Rapid administration (A) of intravenous cephalosporins can potentiate vessel irritation and increase the risk of thrombophlebitis. (C) is not necessary to initiate an alternative IV site. Although aspirin has antiinflammatory actions, (D) is not indicated. Correct Answer: B

A female client asks the nurse to find someone who can translate into her native language her concerns about a treatment. Which action should the nurse take? A. Explain that anyone who speaks her language can answer her questions. B. Provide a translator only in an emergency situation. C. Ask a family member or friend of the client to translate. D. Request and document the name of the certified translator.

A certified translator should be requested to ensure the exchanged information is reliable and unaltered. To adhere to legal requirements in some states, the name of the translator should be documented (D). Client information that is translated is private and protected under HIPAA rules, so (A) is not the best action. Although an emergency situation may require extenuating circumstances (B), a translator should be provided in most situations. Family members may skew information and not translate the exact information, so (C) is not preferred. Correct Answer: D

Inform the family that death is imminent.

A client in hospice care develops audible gurgling sounds on inspiration. Which nursing action has the highest priority?

Take measures to promote as much comfort as possible.

A client is admitted to the hospital with intractable pain. What instruction should the nurse provide the unlicensed assistive personnel (UAP) who is preparing to assist this client with a bed bath?

Wet to moist dressing.

A client is admitted with a stage four pressure ulcer that has a black, hardened surface and a light-pink wound bed with a malodorous green drainage. Which dressing is best for the nurse to use first?

Advise the client to sit on the side of the bed for a few minutes before standing again.

A client who has been on bedrest for several days now has a prescription to progress activity as tolerated. When the nurse assists the client out of bed for the first time, the client becomes dizzy. What action should the nurse implement?

Biofeedback allows the client to control involuntary responses to promote peripheral vasodilation.

A client with Raynaud's disease asks the nurse about using biofeedback for self-management of symptoms. What response is best for the nurse to provide?

Serum albumin.

A client with chronic renal disease is admitted to the hospital for evaluation prior to a surgical procedure. Which laboratory test indicates the client's protein status for the longest length of time?

Which technique is most important for the nurse to implement when performing a physical assessment?

A consistent, systematic approach.

Sensory pattern, area, intensity, and nature of the pain.

A female client who has breast cancer with metastasis to the liver and spine is admitted with constant, severe pain despite around-the-clock use of oxycodone (Percodan) and amitriptyline (Elavil) for pain control at home. During the admission assessment, which information is most important for the nurse to obtain?

The nurse is assessing the nutritional status of several clients. Which client has the greatest nutritional need for additional intake of protein? A. A college-age track runner with a sprained ankle. B. A lactating woman nursing her 3-day-old infant. C. A school-aged child with Type 2 diabetes. D. An elderly man being treated for a peptic ulcer.

A lactating woman (B) has the greatest need for additional protein intake. (A, C, and D) are all conditions that require protein, but do not have the increased metabolic protein demands of lactation. Correct Answer: B

Combination of plant proteins to provide essential amino acids.

A male client with an infected wound tells the nurse that he follows a macrobiotic diet. Which type of foods should the nurse recommend that the client select from the hospital menu?

0800; 1200; 1600; 2000

A medication is prescribed to be given QID. What schedule should the nurse use to administer this prescription?

1. The nurse understands that which information is correct about osteoarthritis?

A noninflammatory condition involving formation of new joint tissue in response to cartilage destruction

Examine one's own culturally based values, beliefs, attitudes, and practices.

A nurse is becoming increasingly frustrated by the family members' efforts to participate in the care of a hospitalized client. What action should the nurse implement to cope with these feelings of frustration?

Withhold the administration of the suppository until contacting the healthcare provider.

A nurse is preparing to insert a rectal suppository and observes a small amount of rectal bleeding. What action should the nurse implement?

A postoperative client will need to perform daily dressing changes after discharge. Which outcome statement best demonstrates the client's readiness to manage his wound care after discharge? The client A. asks relevant questions regarding the dressing change. B. states he will be able to complete the wound care regimen. C. demonstrates the wound care procedure correctly. D. has all the necessary supplies for wound care.

A return demonstration of a procedure (C) provides an objective assessment of the client's ability to perform a task, while (A and B) are subjective measures. (D) is important, but is less of a priority prior to discharge than the nurse's assessment of the client's ability to complete the wound care. Correct Answer: C

Nurse who puts meds in her uniform pocket to deliver to clients confides that after arriving home, she found a hydrocodone tablet in her pocket. Which possible outcome of this situation should be the nurse's greatest concern?

Accused of diversion

A female nurse who sometimes tries to save time by putting medications in her uniform pocket to deliver to clients, confides that after arriving home she found a hydrocodone (Vicodin) tablet in her pocket. Which possible outcome of this situation should be the nurse's greatest concern?

Accused of diversion.

A female nurse who sometimes tries to save time by putting medications in her uniform pocket to deliver to clients, confides that after arriving home she found a hydrocodone (Vicodin) tablet in her pocket. Which possible outcome of this situation should be the nurse's greatest concern? Accused of diversion. Reported for stealing. Reported for a HIPAA violation. Accused of unprofessional conduct.

Accused of diversion.

During a physical assessment, a female client begins to cry. Which action is best for the nurse to take? A. Request another nurse to complete the physical assessment. B. Ask the client to stop crying and tell the nurse what is wrong. C. Acknowledge the client's distress and tell her it is all right to cry. D. Leave the room so that the client can be alone to cry in private.

Acknowledging the client's distress and giving the client the opportunity to verbalize her distress (C) is a supportive response. (A, B, and D) are not supportive and do not facilitate the client's expression of feelings. Correct Answer: C

The nurse is caring for a client who is weak from inactivity because of a 2-week hospitalization. In planning care for the client, the nurse should include which ROM exercises?

Active ROM exercises to both arms & legs 2-3 times daily

The nurse is caring for a client who is weak from inactivity because of a 2-week hospitalization. In planning care for the client, the nurse should include which range of motion (ROM) exercises?

Active ROM exercises to both arms and legs two or three times a day.

The nurse is caring for a client who is weak from inactivity because of a 2-week hospitalization. In planning care for the client, the nurse should include which range of motion (ROM) exercises? Passive ROM exercises to all joints on all extremities four times a day. Active ROM exercises to both arms and legs two or three times a day. Active ROM exercises with weights twice a day with 20 repetitions each. Passive ROM exercises to the point of resistance and slightly beyond.

Active ROM exercises to both arms and legs two or three times a day.

When the nurse enters a client's room to do an initial assessment, the client shouts, "Get out of my room! I'm tired of being bothered!" How should the nurse respond? Passive ROM exercises to all joints on all extremities four times a day. Active ROM exercises to both arms and legs two or three times a day. Active ROM exercises with weights twice a day with 20 repetitions each. Passive ROM exercises to the point of resistance and slightly beyond.

Active ROM exercises to both arms and legs two or three times a day. Rationale Active, rather than passive, ROM is best to restore strength and (B) is an effective schedule. Passive ROM 4 times a day (A) is not as beneficial for the client as (B). With weights (C), the client may fatigue quickly and develop muscle soreness. ROM is not performed beyond the point of resistance or pain (D) because of the risk of damage to underlying structures.

A client who has been on bedrest for several days now has a prescription to progress activity as tolerated. When the nurse assists the client out of bed for the first time, the client becomes dizzy. What action should the nurse implement?

Advise the client to sit on the side of the bed for a few minutes before standing again.

A client who has been on bedrest for several days now has a prescription to progress daily activity as tolerated. When nurse assists client out of bed for the first time, the client becomes dizzy. What action should the nurse implement?

Advise the client to sit on the side of the bed for a few minutes before standing again.

A client with acute hemorrhagic anemia is to receive four units of packed RBCs (red blood cells) as rapidly as possible. Which intervention is most important for the nurse to implement? A. Obtain the pre-transfusion hemoglobin level. B. Prime the tubing and prepare a blood pump set-up. C. Monitor vital signs q15 minutes for the first hour. D. Ensure the accuracy of the blood type match.

All interventions should be implemented prior to administering blood, but (D) has the highest priority. Any time blood is administered, the nurse should ensure the accuracy of the blood type match in order to prevent a possible hemolytic reaction. Correct Answer: D

A client with pneumonia has a decrease in oxygen saturation from 94% to 88% while ambulating. Based on these findings, which intervention should the nurse implement first? A. Assist the ambulating client back to the bed. B. Encourage the client to ambulate to resolve pneumonia. C. Obtain a prescription for portable oxygen while ambulating. D. Move the oximetry probe from the finger to the earlobe.

An oxygen saturation below 90% indicates inadequate oxygenation. First, the client should be assisted to return to bed (A) to minimize oxygen demands. Ambulation increases aeration of the lungs to prevent pooling of respiratory secretions, but the client's activity at this time is depleting oxygen saturation of the blood, so (B) is contraindicated. Increased activity increases respiratory effort, and oxygen may be necessary to continue ambulation (C), but first the client should return to bed to rest. Oxygen saturation levels at different sites should be evaluated after the client returns to bed (D). Correct Answer: A

Prior to administering newly prescribed medication, nurse reviews adverse effects & determines priority risks to the client. While performing this action, nurse is engaged in which step of the nursing process?

Analysis

61.The daughter of an older woman who became depressed following the death of her husband asks, "My mother was always well-adjusted until my father died. Will she tend to be sick from now on?" Which response is best for the nurse to provide? A. She is almost sure to be less able to adapt than before. B. It's highly likely that she will recover and return to her pre-illness state. C. If you can interest her in something besides religion, it will help her stay well. D. Cultural strains contribute to each woman's tendencies for recurrences of depression.

Analysis of behavior patterns using Erikson's framework can identify age-appropriate or arrested development of normal interpersonal skills. Erikson describes the successful resolution of a developmental crisis in the later years (older than 65-years) to include the achievement of a sense of integrity and fulfillment, wisdom, and a willingness to face one's own mortality and accept the death of others (B). Depression is a component of normal grieving, and (A) does not represent susceptible adaptation to the developmental crisis of an older adult, Integrity vs despair. (C and D) are judgmental and not therapeutic. Correct Answer: B

Prior to administering a newly prescribed medication to a client, the nurse reviews the adverse effects of the medication listed in a drug reference guide and determines the priority risks to the client. While performing this action, the nurse is engaged in which step of the nursing process?

Analysis.

Prior to administering a newly prescribed medication to a client, the nurse reviews the adverse effects of the medication listed in a drug reference guide and determines the priority risks to the client. While performing this action, the nurse is engaged in which step of the nursing process? Assessment. Analysis. Implementation. Evaluation.

Analysis.

15. Which post-operative prescription should the nurse implement first?

Apply oxygen 2 L via nasal cannula

What is the rationale in using the nursing process in planning care for clients?

As a tool to organize thinking & clinical decision making about clients' healthcare needs

What is the rationale for using the nursing process in planning care for clients?

As a tool to organize thinking and clinical decision making about clients' healthcare needs.

Client demonstrates positive Chvostek's sign. What action should the nurse take?

Ask the client about numbness or tingling in the hands

A client is demonstrating a positive Chvostek's sign. What action should the nurse take? Observe the client's pupil size and response to light. Ask the client about numbness or tingling in the hands. Assess the client's serum potassium level. Restrict dietary intake of calcium-rich foods.

Ask the client about numbness or tingling in the hands. Rationale A positive Chvostek's sign is an indication of hypocalcemia, so the client should be assessed for the subjective symptoms of hypocalcemia, such as numbness or tingling of the hands (B) or feet. (A and C) are unrelated assessment data. (D) is contraindicated because the client is hypocalcemic and needs additional dietary calcium.

Male client with AIDS develops cryptococcal meningitis & tells the nurse he does not want to be resuscitated if his breathing stops. What action should the nurse implement?

Ask the client if his decision has been discussed with his HCP.

A male client with acquired immunodeficiency syndrome (AIDS) develops cryptococcal meningitis and tells the nurse he does not want to be resuscitated if his breathing stops. What action should the nurse implement?

Ask the client if this decision has been discussed with his healthcare provider.

A male client with acquired immunodeficiency syndrome (AIDS) develops cryptococcal meningitis and tells the nurse he does not want to be resuscitated if his breathing stops. What action should the nurse implement? Document the client's request in the medical record. Ask the client if this decision has been discussed with his healthcare provider. Inform the client that a written, notarized advance directive, is required to withhold resuscitation efforts. Advise the client to designate a person to make healthcare decisions when the client is unable to do so.

Ask the client if this decision has been discussed with his healthcare provider.

Nurse encounters slight resistance when inserting tubing into a client's rectum for a tap water enema. What action should the nurse implement?

Ask the client to relax & twist the tube gently through the sphincter

The nurse encounters resistance when inserting the tubing into a client's rectum for a tap water enema. What action should the nurse implement?

Ask the client to relax and run a small amount of fluid into the rectum.

The nurse encounters resistance when inserting the tubing into a client's rectum for a tap water enema. What action should the nurse implement? Withdraw the tube and apply additional lubricant to the tube. Encourage the client to bear down and continue to insert the tube. Remove the tube and check the client for a fecal impaction. Ask the client to relax and run a small amount of fluid into the rectum.

Ask the client to relax and run a small amount of fluid into the rectum. Rationale If resistance is encountered during the initial insertion of an enema tube, the client should be instructed to relax while a small amount of solution runs through the tube into the rectum (D) to promote dilation. (A) is unlikely to resolve the problem. (B) may cause injury. (C) should not be implemented until other, less invasive actions, such as (D) have been taken.

As the nurse prepares the equipment to be used to start an IV on a 4y/o boy in the treatment room, he cries continuously. What intervention should the nurse implement?

Ask the mother to be present to soothe the child

As the nurse prepares the equipment to be used to start an IV on a 4-year-old boy in the treatment room, he cries continuously. What intervention should the nurse implement?

Ask the mother to be present to soothe the child.

What action by the nurse demonstrates culturally sensitive care?

Asks permission before touching a client

What action by the nurse demonstrates culturally sensitive care?

Asks permission before touching a client.

What action by the nurse demonstrates culturally sensitive care? Asks permission before touching a client. Avoids questions about male-female relationships. Explains the differences between Western medical care and cultural folk remedies. Applies knowledge of a cultural group unless a client embraces Western

Asks permission before touching a client.

What action by the nurse demonstrates culturally sensitive care? Asks permission before touching a client. Avoids questions about male-female relationships. Explains the differences between Western medical care and cultural folk remedies. Applies knowledge of a cultural group unless a client embraces Western customs.

Asks permission before touching a client. Rationale Physical contact, such as touching the head, in some cultures is a sign of respect, whereas in others, it is strictly forbidden. So asking permission before touching a client demonstrates culturally sensitive care.

A signed consent form indicated a client should have an electromyogram, by a myelogram was performed instead. Thought the myelogram revealed the cause of the client's back pain, the client filed a lawsuit against the nurse & HCP for performing the incorrect procedure. The court is likely to rule in favor of the plaintiff because these events represent which infraction?

Assault & battery with deliberate intent to deviate from the consent form

A signed consent form indicated a client should have an electromyogram, but a myelogram was performed instead. Though the myelogram revealed the cause of the client's back pain, which was subsequently treated, the client filed a lawsuit against the nurse and healthcare provider for performing the incorrect procedure. The court is likely to rule in favor of the plaintiff because these events represent what infraction?

Assault and battery with deliberate intent to deviate from the consent form.

A signed consent form indicated a client should have an electromyogram, but a myelogram was performed instead. Though the myelogram revealed the cause of the client's back pain, which was subsequently treated, the client filed a lawsuit against the nurse and healthcare provider for performing the incorrect procedure. The court is likely to rule in favor of the plaintiff because these events represent what infraction? A quasi-intentional tort because a similar mistake can happen to anyone. Failure to respect client autonomy to choose based on intentional tort law. Assault and battery with deliberate intent to deviate from the consent form. An unintentional tort because the client benefited from having the myelogram.

Assault and battery with deliberate intent to deviate from the consent form.

Which intervention is most important for the nurse to implement for a male client who is experiencing urinary retention? Apply a condom catheter. Apply a skin protectant. Encourage increased fluid intake. Assess for bladder distention.

Assess for bladder distention.

Client with nursing diagnosis of "Spiritual distress r/t loss of hope secondary to impending death." What intervention is best for the nurse to implement when caring for this client?

Assist & support the client in establishing short-term goals. Rationale: Hopefulness is necessary to sustain a meaningful existence, even close to death.

A client has a nursing diagnosis of, "Spiritual distress related to a loss of hope, secondary to impending death." What intervention is best for the nurse to implement when caring for this client?

Assist and support the client in establishing short-term goals.

A client has a nursing diagnosis of, "Spiritual distress related to a loss of hope, secondary to impending death." What intervention is best for the nurse to implement when caring for this client? Help the client to accept the final stage of life. Assist and support the client in establishing short-term goals. Encourage the client to make future plans, even if they are unrealistic. Instruct the client's family to focus on positive aspects of the client's life.

Assist and support the client in establishing short-term goals.

The nurse is teaching a client with numerous allergies how to avoid allergens. Which instruction should be included in this teaching plan? Avoid any types of sprays, powders, and perfumes. Wearing a mask while cleaning will not help to avoid allergens. Purchase any type of clothing, but be sure it is washed before wearing it. Pollen count is related to hay fever, not to allergens.

Avoid any types of sprays, powders, and perfumes.

On admission, a client presents a signed living will that includes a Do Not Resuscitate (DNR) prescription. When the client stops breathing, the nurse performs cardiopulmonary resuscitation (CPR) and successfully revives the client. What legal issues could be brought against the nurse? Assault. Battery. Malpractice. False imprisonment.

Battery

After completing an assessment and determining that a client has a problem, which action should the nurse perform next? A. Determine the etiology of the problem. B. Prioritize nursing care interventions. C. Plan appropriate interventions. D. Collaborate with the client to set goals.

Before planning care, the nurse should determine the etiology, or cause, of the problem (A), because this will help determine (B, C, and D). Correct Answer: A

The charge nurse observes a UAP bending at the waist to lift a 20lb box. What instruction should the charge nurse provide?

Bend at the knees when lifting heavy objects

The charge nurse observes an unlicensed assistive personnel (UAP) bending at the waist to lift a 20-pound box of medical supplies off the treatment room floor. What instruction should the charge nurse provide to the UAP?

Bend at the knees when lifting heavy objects.

12. Which ethical principle supports the decision to proceed with the emergency surgery when the client cannot give informed consent?

Beneficence

Client with Raynaud's disease asks the nurse about using biofeedback for self-management of symptoms. What response is best for the nurse to provide?

Biofeedback allows the client to control involuntary responses to promote peripheral vasodilation

A client with Raynaud's disease asks the nurse about using biofeedback for self-management of symptoms. What response is best for the nurse to provide?

Biofeedback allows the client to control involuntary responses to promote peripheral vasodilation.

A client with Raynaud's disease asks the nurse about using biofeedback for self-management of symptoms. What response is best for the nurse to provide? The responses to biofeedback have not been well established and may be a waste of time and money. Biofeedback requires extensive training to retrain voluntary muscles, not involuntary responses. Although biofeedback is easily learned, it is most often used to manage exacerbation of symptoms. Biofeedback allows the client to control involuntary responses to promote peripheral vasodilation.

Biofeedback allows the client to control involuntary responses to promote peripheral vasodilation. Rationale Biofeedback involves the use of various monitoring devices that help people become more aware and able to control their own physiologic responses, such as heart rate, body temperature, muscle tension, and brain waves. (D) is an accurate statement concerning its use for clients with Raynaud's disease. (A, B, and C) do not provide correct information about biofeedback.

3. Which symptom should Mrs. Weil report immediately while taking a non-steroidal anti-inflammatory drug (NSAID)?

Black, tarry stools

A client who is a Jehovah's Witness is admitted to the nursing unit. Which concern should the nurse have for planning care in terms of the client's beliefs? A. Autopsy of the body is prohibited. B. Blood transfusions are forbidden. C. Alcohol use in any form is not allowed. D. A vegetarian diet must be followed.

Blood transfusions are forbidden (B) in the Jehovah's Witness religion. Judaism prohibits (A). Buddhism forbids the use of (C) and drugs. Many of these sects are vegetarian (D), but the direct impact on nursing care is (B). Correct Answer: B

A client who is a Jehovah's Witness is admitted to the nursing unit. Which concern should the nurse have for planning care in terms of the client's beliefs? Autopsy of the body is prohibited. Blood transfusions are forbidden. Alcohol use in any form is not allowed. A vegetarian diet must be followed.

Blood transfusions are forbidden.

Which assessment data would provide the most accurate determination of proper placement of a nasogastric tube? A. Aspirating gastric contents to assure a pH value of 4 or less. B. Hearing air pass in the stomach after injecting air into the tubing. C. Examining a chest x-ray obtained after the tubing was inserted. D. Checking the remaining length of tubing to ensure that the correct length was inserted.

Both (A and B) are methods used to determine proper placement of the NG tubing. However, the best indicator that the tubing is properly placed is (C). (D) is not an indicator of proper placement. Correct Answer: C

A young mother of three children complains of increased anxiety during her annual physical exam. What information should the nurse obtain first? A. Sexual activity patterns. B. Nutritional history. C. Leisure activities. D. Financial stressors.

Caffeine, sugars, and alcohol can lead to increased levels of anxiety, so a nutritional history (C) should be obtained first so that health teaching can be initiated if indicated. (A and C) can be used for stress management. Though (D) can be a source of anxiety, a nutritional history should be obtained first. Correct Answer: B

24. While the nurse is awaiting the return call from the physician, Susan complains of sudden shortness of breath and chest pain. What is the priority action?

Call for a Rapid Response Team

The nurse is preparing to give a client with dehydration IV fluids delivered at a continuous rate of 175 mL/hr. Which infusion device should the nurse use?

Cassette infusion pump

The nurse is preparing to give a client dehydration IV fluids delivered at a continuous rate of 175 ml/hour. Which infusion device should the nurse use?

Cassette infusion pump.

The nurse is preparing to give a client dehydration IV fluids delivered at a continuous rate of 175 ml/hour. Which infusion device should the nurse use? Portable syringe pump. Cassette infusion pump. Volumetric controller. Nonvolumetric controller.

Cassette infusion pump.

What intervention should the nurse include in the care plan for a client who is being treated with an Unna's paste boot for leg ulcers due to chronic venous insufficiency?

Check capillary refill of toes on lower extremity with Unna's paste boot Rationale: Boot becomes rigid after it dries, so it is important to check distally for adequate circulation. No bandage should be put under it. Should be applied from foot & wrapped towards knee. Acts as a sterile dressing & should not be removed q8h. Weekly removal is reasonable.

What intervention should the nurse include in the plan of care for a client who is being treated with an Unna's paste boot for leg ulcers due to chronic venous insufficiency?

Check capillary refill of toes on lower extremity with Unna's paste boot.

Which statement best describes durable POA for healthcare?

Client signs a document that designates another person to make legally binding healthcare decisions if the client is unable to do so

During the admission interview, which technique is most efficient for the nurse to use when obtaining information about signs and symptoms of a client's primary health problem? Restatement of responses. Open-ended questions. Closed-ended questions. Problem-seeking responses.

Closed-ended questions.

Client with an infected wound tells nurse he follows a macrobiotic diet. What foods should the nurse recommend?

Combination of plant proteins to provide essential amino acids

A male client with an infected wound tells the nurse that he follows a macrobiotic diet. Which type of foods should the nurse recommend that the client select from the hospital menu?

Combination of plant proteins to provide essential amino acids.

A male client with an infected wound tells the nurse that he follows a macrobiotic diet. Which type of foods should the nurse recommend that the client select from the hospital menu? Low fat and low sodium foods. Combination of plant proteins to provide essential amino acids. Limited complex carbohydrates and fiber. Increased amount of vitamin C and beta carotene rich foods.

Combination of plant proteins to provide essential amino acids.

A client with chronic kidney disease (CKD) selects a scrambled egg for his breakfast. Which action should the nurse take? Commend the client for selecting a high biologic value protein. Remind the client that protein in the diet should be avoided. Suggest that the client also select orange juice, to promote absorption. Encourage the client to attend classes on dietary management of CKD. Rationale

Commend the client for selecting a high biologic value protein.

The nurse notices that the Hispanic parents of a toddler who returns from surgery offer the child only the broth that comes on the clear liquid tray. Other liquids, including gelatin, popsicles, and juices, remain untouched. What explanation is most appropriate for this behavior? A. The belief is held that the "evil eye" enters the child if anything cold is ingested. B. After surgery the child probably has refused all foods except broth. C. Eating broth strengthens the child's innate energy called "chi." D. Hot remedies restore balance after surgery, which is considered a "cold" condition.

Common parental practices and health beliefs among Hispanic, Chinese, Filipino, and Arab cultures classify diseases, areas of the body, and illnesses as "hot" or "cold" and must be balanced to maintain health and prevent illness. The perception that surgery is a "cold" condition implies that only "hot" remedies, such as soup, should be used to restore the healthy balance within the body, so (D) is the correct interpretation. (A, B, and C) are not correct interpretations of the noted behavior. "Chi" is a Chinese belief that an innate energy enters and leaves the body via certain locations and pathways and maintains health. The "evil eye," or "mal ojo," is believed by many cultures to be related to the balance of health and illness but is unrelated to dietary practice. Correct Answer: D

The nurse is completing the care plan for a client who is admitted for BPH. Which data should the nurse document as a subjective finding?

Complains of inability to empty bladder

The nurse is completing the plan of care for a client who is admitted for benign prostatic hypertrophy. Which data should the nurse document as a subjective findings?

Complains of inability to empty bladder.

The nurse is completing the plan of care for a client who is admitted for benign prostatic hypertrophy. Which data should the nurse document as a subjective findings? Complains of inability to empty bladder. Temperature of 99.8 ??F and pulse of 108. Post-voided residual volume of 750 ml. Specimen collection for culture and sensitivity.

Complains of inability to empty bladder.

What action should the nurse implement when adding sterile liquids to a sterile field?

Consider the sterile field contaminated if it becomes wet during the procedure

What action should the nurse implement when adding sterile liquids to a sterile field?

Consider the sterile field contaminated if it becomes wet during the procedure.

Which technique is most important for a nurse to implement when performing a physical assessment?

Consistent, systematic approach

Client who has moderate, persistent, chronic neuropathic pain d/t diabetic neuropathy takes gabapentin & ibuprofen daily. If Step 2 of the WHO pain relief ladder is prescribed, which drug protocol should be implemented?

Continue gabapentin

A client who has moderate, persistent, chronic neuropathic pain due to diabetic neuropathy takes gabapentin (Neurontin) and ibuprofen (Motrin, Advil) daily. If Step 2 of the World Health Organization (WHO) pain relief ladder is prescribed, which drug protocol should be implemented?

Continue gabapentin.

A client who has moderate, persistent, chronic neuropathic pain due to diabetic neuropathy takes gabapentin (Neurontin) and ibuprofen (Motrin, Advil) daily. If Step 2 of the World Health Organization (WHO) pain relief ladder is prescribed, which drug protocol should be implemented? Continue gabapentin. Discontinue ibuprofen. Add aspirin to the protocol. Add oral methadone to the protocol.

Continue gabapentin.

When assessing client with nursing dx of fluid volume deficit, the nurse notes that the client's skin over sternum "tents" when gently pinched. Which action should the nurse implement?

Continue the planned nursing interventions to restore the client's fluid volume

When assessing a client with a nursing diagnosis of fluid volume deficit, the nurse notes that the client's skin over the sternum "tents" when gently pinched. Which action should the nurse implement?

Continue the planned nursing interventions to restore the client's fluid volume.

When assessing a client with a nursing diagnosis of fluid volume deficit, the nurse notes that the client's skin over the sternum "tents" when gently pinched. Which action should the nurse implement? Confirm the finding by further assessing the client for jugular vein distention. Offer the client high protein snacks between regularly scheduled mealtimes. Continue the planned nursing interventions to restore the client's fluid volume. Change the plan of care to include a nursing diagnosis of impaired skin integrity.

Continue the planned nursing interventions to restore the client's fluid volume. Rationale Skin turgor is assessed by pinching the skin and observing for tenting. This finding confirms the diagnosis of fluid volume deficit, so the nurse should continue interventions to restore the client's fluid volume (C). Jugular vein distention (A) is a sign of fluid volume overload. High protein snacks (B) will not resolve the fluid volume deficit. Changes in the client's skin integrity are not evident (D).

When conducting an admission assessment, the nurse should ask the client about the use of complimentary healing practices. Which statement is accurate regarding the use of these practices? A. Complimentary healing practices interfere with the efficacy of the medical model of treatment. B. Conventional medications are likely to interact with folk remedies and cause adverse effects. C. Many complimentary healing practices can be used in conjunction with conventional practices. D. Conventional medical practices will ultimately replace the use of complimentary healing practices.

Conventional approaches to health care can be depersonalizing and often fail to take into consideration all aspects of an individual, including body, mind, and spirit. Often complimentary healing practices can be used in conjunction with conventional medical practices (C), rather than interfering (A) with conventional practices, causing adverse effects (B), or replacing conventional medical care (D). Correct Answer: C

42.The nurse notes that a client consistently coughs while eating and drinking. Which nursing diagnosis is most important for the nurse include in this client's plan of care? A. Ineffective breathing pattern. B. Impaired gas exchange. C. Risk for aspiration. D. Ineffective airway clearance.

Coughing during or after meals is a manifestation of dysphagia, or difficulty swallowing, which places the client at risk for aspiration (C). Dysphagia can lead to aspiration pneumonia, but the client is not currently exhibiting any symptoms of breathing difficulty (A) or impaired gas exchange (B). Although (D) may be related to an ineffective cough, the client's coughing is an effective response when solids or liquids are taken orally. Correct Answer: C

A hospitalized male client is receiving nasogastric tube feedings via a small-bore tube and a continuous pump infusion. He reports that he had a bad bout of severe coughing a few minutes ago, but feels fine now. What action is best for the nurse to take? A. Record the coughing incident. No further action is required at this time. B. Stop the feeding, explain to the family why it is being stopped, and notify the healthcare provider. C. After clearing the tube with 30 ml of air, check the pH of fluid withdrawn from the tube. D. Inject 30 ml of air into the tube while auscultating the epigastrium for gurgling.

Coughing, vomiting, and suctioning can precipitate displacement of the tip of the small bore feeding tube upward into the esophagus, placing the client at increased risk for aspiration. Checking the sample of fluid withdrawn from the tube (after clearing the tube with 30 ml of air) for acidic (stomach) or alkaline (intestine) values is a more sensitive method for these tubes, and the nurse should assess tube placement in this way prior to taking any other action (C). (A and B) are not indicated. The auscultating method (D) has been found to be unreliable for small-bore feeding tubes. Correct Answer: C

Nurse is providing passive ROM exercises to hip & knee for unconscious client. After supporting the client's knee with one hand, what action should the nurse take?

Cradle the client's heel

The nurse is providing passive range of motion (ROM) exercises to the hip and knee for a client who is unconscious. After supporting the client's knee with one hand, what action should the nurse take next?

Cradle the client's heel.

The nurse is providing passive range of motion (ROM) exercises to the hip and knee for a client who is unconscious. After supporting the client's knee with one hand, what action should the nurse take next? Raise the bed to a comfortable working level. Bend the client's knee. Move the knee toward the chest as far as it will go. Cradle the client's heel.

Cradle the client's heel.

The nurse is providing passive range of motion (ROM) exercises to the hip and knee for a client who is unconscious. After supporting the client's knee with one hand, what action should the nurse take next? Raise the bed to a comfortable working level. Bend the client's knee. Move the knee toward the chest as far as it will go. Cradle the client's heel.

Cradle the client's heel. Rationale Passive ROM exercise for the hip and knee is provided by supporting the joints of the knee and ankle (D) and gently moving the limb in a slow, smooth, firm but gentle manner. (A) should be done before the exercises are begun to prevent injury to the nurse and client. (B) is carried out after both joints are supported. After the knee is bent, then the knee is moved toward the chest to the point of resistance (C) two or three times.

While the nurse is administering a bolus feeding to a client via NG tube, the client begins to vomit. What action should the nurse implement first?

D/c administration of the bolus feeding

During the initial morning assessment, a male client denies dysuria but reports that his urine appears dark amber. Which intervention should the nurse implement? A. Provide additional coffee on the client's breakfast tray. B. Exchange the client's grape juice for cranberry juice. C. Bring the client additional fruit at mid-morning. D. Encourage additional oral intake of juices and water.

Dark amber urine is characteristic of fluid volume deficit, and the client should be encouraged to increase fluid intake (D). Caffeine, however, is a diuretic (A), and may worsen the fluid volume deficit. Any type of juice will be beneficial (B), since the client is not dysuric, a sign of an urinary tract infection. The client needs to restore fluid volume more than solid foods (C). Correct Answer: D

During a visit to the outpatient clinic, the nurse assesses a client with severe osteoarthritis using a goniometer. Which finding should the nurse expect to measure? Adequate venous blood flow to the lower extremities. Estimated amount of body fat by an underarm skinfold. Degree of flexion and extension of the client's knee joint. Change in the circumference of the joint in centimeters.

Degree of flexion and extension of the client's knee joint.

In evaluating client care, which action should the nurse take first?

Determine if expected outcomes of care were achieved

In evaluating client care, which action should the nurse take first?

Determine if the expected outcomes of care were achieved.

14. Which action should the nurse take first?

Determine the respiratory rate

A 75y/o client who has a history of end stage renal failure & advancing lung cancer, recently had a stroke. 2 days ago, the HCP d/c the client's dialysis treatments, stating that death is inevitable, but the client is disoriented & will not sign a DNR directive. What is the priority nursing intervention?

Determine who is legally empowered to make decisions

A 75-year-old client who has a history of end stage renal failure and advanced lung cancer, recently had a stroke. Two days ago the healthcare provider discontinued the client's dialysis treatments, stating that death is inevitable, but the client is disoriented and will not sign a DNR directive. What is the priority nursing intervention?

Determine who is legally empowered to make decisions.

A 75-year-old client who has a history of end stage renal failure and advanced lung cancer, recently had a stroke. Two days ago the healthcare provider discontinued the client's dialysis treatments, stating that death is inevitable, but the client is disoriented and will not sign a DNR directive. What is the priority nursing intervention? Review the client's most recent laboratory reports. Refer the client and family members for hospice care. Notify the hospital ethics committee of the client situation. Determine who is legally empowered to make decisions.

Determine who is legally empowered to make decisions.

An African-American grandmother tells the nurse that her 4-year-old grandson is suffering with "miseries." Based on this statement, which focused assessment should the nurse conduct? A. Inquire about the source and type of pain. B. Examine the nose for congestion and discharge. C. Take vital signs for temperature elevation. D. Explore the abdominal area for distension.

Different cultural groups often have their own terms for health conditions. African-American clients may refer to pain as "the miseries. " Based on understanding this term, the nurse should conduct a focused assessment on the source and type of pain (A). (B, C, and D) are important, but do not focus on "miseries" (pain). Correct Answer: A

While the nurse is administering a bolus feeding to a client via nasogastric tube, the client begins to vomit. What action should the nurse implement first? Discontinue the administration of the bolus feeding. Auscultate the client's breath sounds bilaterally. Elevate the head of the bed to a high Fowler's position. Administer a PRN dose of a prescribed antiemetic.

Discontinue the administration of the bolus feeding

While the nurse is administering a bolus feeding to a client via nasogastric tube, the client begins to vomit. What action should the nurse implement first?

Discontinue the administration of the bolus feeding.

The nurse is developing a plan of care for a client with dementia. Which feature of confusion in the elderly is accurate? Bewilderment is to be expected, and progresses with age. Disorientation often follows relocation to new surroundings. Uncertainty is a result of irreversible brain pathology. Being perplexed can be prevented with adequate sleep.

Disorientation often follows relocation to new surroundings.

Before administering a client's medication, the nurse assesses a change in the client's condition & withholds the medication until consulting with the HCP. The dose is changed & the nurse administers the new Rx 1 hour later than originally scheduled. What action should the nurse implement in response to this situation?

Document the events that occurred in the nurses' notes.

Before administering a client's medication, the nurse assesses a change in the client's condition and decides to withhold the medication until consulting with the healthcare provider. After consultation with the healthcare provider, the dose of the medication is changed and the nurse administers the newly prescribed dose an hour later than the originally scheduled time. What action should the nurse implement in response to this situation? Notify the charge nurse that a medication error occurred. Submit a medication variance report to the supervisor. Document the events that occurred in the nurses' notes. Discard the original medication administration record.

Document the events that occurred in the nurses' notes.

In assessing a client's femoral pulse, the nurse must use deep palpation to feel the pulsation when the client is in supine position. What action should the nurse implement?

Document the presence & volume of the pulse palpated

In assessing a client's femoral pulse, the nurse must use deep palpation to feel the pulsation while the client is in a supine position. What action should the nurse implement?

Document the presence and volume of the pulse palpated.

AN older client with RA is complaining of severe joint pain that is caused by weight of the linen on her legs. What action should the nurse implement first?

Drape the sheets over the footboard of the bed

An older female client with rheumatoid arthritis is complaining of severe joint pain that is caused by the weight of the linen on her legs. What action should the nurse implement first?

Drape the sheets over the footboard of the bed.

When making bed of a client who needs a bed cradle, which action should the nurse include?

Drape the top sheet & covers loosely over the bed cradle Rationale: Bed cradle is used to keep top bed linens off of the client.

When making the bed of a client who needs a bed cradle, which action should the nurse include?

Drape the top sheet and covers loosely over the bed cradle.

When making the bed of a client who needs a bed cradle, which action should the nurse include? Teach the client to call for help before getting out of bed. Keep both the upper and lower side rails in a raised position. Keep the bed in the lowest position while changing the sheets. Drape the top sheet and covers loosely over the bed cradle.

Drape the top sheet and covers loosely over the bed cradle.

When making the bed of a client who needs a bed cradle, which action should the nurse include? Teach the client to call for help before getting out of bed. Keep both the upper and lower side rails in a raised position. Keep the bed in the lowest position while changing the sheets. Drape the top sheet and covers loosely over the bed cradle.

Drape the top sheet and covers loosely over the bed cradle. Rationale A bed cradle is used to keep the top bedclothes off the client, so the nurse should drape the top sheet and covers loosely over the cradle (D). A client using a bed cradle may still be able to ambulate independently (A) and does not require raised side rails (B). (C) causes the nurse to use poor body mechanics.

11. What is the priority action in preparation for Susan's surgery?

Draw blood for type and cross match

The nurse is preparing to irrigate a client's indwelling urinary catheter using an open technique. What action should the nurse take after applying gloves? Empty the client's urinary drainage bag. Draw up the irrigating solution into the syringe. Secure the client's catheter to the drainage tubing. Use aseptic technique to instill the

Draw up the irrigating solution into the syringe

The nurse is preparing to irrigate a client's indwelling urinary catheter using an open technique. What action should the nurse take after applying gloves?

Draw up the irrigating solution into the syringe.

The nurse is preparing to irrigate a client's indwelling urinary catheter using an open technique. What action should the nurse take after applying gloves?

Draw up the irrigating solution into the syringe. Rationale: First apply gloves, then draw up the irrigating solution. Syringe is attached to catheter & fluid is instilled, using aseptic technique. Once instilled, catheter should be secured to drainage tubing. Drainage bag can be emptied whenever I&O measurement is indicated

Nurse formulates nursing diagnosis of "ineffective health maintenance related to lack of motivation" for a client with DM2. Which finding supports this nursing diagnosis?

Eats anything & does not think diet makes a difference in health Rationale: Diagnosis is best exemplified in client belief or understanding about diet & health maintenance.

The nurse formulates the nursing diagnosis of, "Ineffective health maintenance related to lack of motivation" for a client with Type 2 diabetes. Which finding supports this nursing diagnosis? Does not check capillary blood glucose as directed. Occasionally forgets to take daily prescribed medication. Cannot identify signs or symptoms of high and low blood glucose. Eats anything and does not think diet makes a difference in health.

Eats anything and does not think diet makes a difference in health

The nurse formulates the nursing diagnosis of, "Ineffective health maintenance related to lack of motivation" for a client with Type 2 diabetes. Which finding supports this nursing diagnosis?

Eats anything and does not think diet makes a difference in health.

10. The nurse understands that which is a correct description of compartment syndrome?

Elevated pressure within a confined myofascial section compromises neurovascular function

A client provides the nurse with information about the reason for seeking care. The nurse realizes that some information about past hospitalizations is missing. How should the nurse obtain this information?

Elicit specific facts about past hospitalizations with direct questions.

A client provides the nurse with information about the reason for seeking care. The nurse realizes that some information about past hospitalizations is missing. How should the nurse obtain this information? Solicit information on hospitalization from the insurance company. Look up previous medical records from archived hospital documents. Ask the client to discuss previous hospitalizations in the last 5 years. Elicit specific facts about past hospitalizations with direct questions.

Elicit specific facts about past hospitalizations with direct questions.

A client provides the nurse with information about the reason for seeking care. The nurse realizes that some information about past hospitalizations is missing. How should the nurse obtain this information? Solicit information on hospitalization from the insurance company. Look up previous medical records from archived hospital documents. Ask the client to discuss previous hospitalizations in the last 5 years. Elicit specific facts about past hospitalizations with direct questions.

Elicit specific facts about past hospitalizations with direct questions. Rationale Direct questions should be used after the client's opening narrative to fill in any details that have been left out or during the review of systems to elicit specific facts about past health problems.

A client provides the nurse with information about the reason for seeking care. Nurse realizes some info about past hospitalizations is missing. How should the nurse obtain this information?

Elicit specific fats about past hospitalizations with direct questions

Which client care activity requires the nurse to wear barrier gloves as required by the protocol for Standard Precautions?

Emptying the urinary catheter drainage bag for a client with Alzheimer's disease

Which client care activity requires the nurse to wear barrier gloves as required by the protocol for Standard Precautions?

Emptying the urinary catheter drainage bag for a client with Alzheimer's disease.

Which client care activity requires the nurse to wear barrier gloves as required by the protocol for Standard Precautions? Removing the empty food tray from a client with a urinary catheter. Washing and combing the hair of a client with a fractured leg in traction. Administering oral medications to a cooperative client with a wound infection. Emptying the urinary catheter drainage bag for a client with Alzheimer's disease.

Emptying the urinary catheter drainage bag for a client with Alzheimer's disease.

A 35y/o client with cancer refuses to allow a nurse to insert an IV for scheduled chemo & states that she's ready to go home to die. What intervention should the nurse initiate?

Evaluate the client's mental status for competence to refuse treatment Rationale: Competent clients have the right to refuse treatment. The nurse cannot document until the HCP is notified of the patient's wishes & a d/c RX is obtained. Advance directives & DNR are not necessary for competent client to refuse care.

A 35-year-old female client with cancer refuses to allow the nurse to insert an IV for a scheduled chemotherapy treatment, and states that she is ready to go home to die. What intervention should the nurse initiate?

Evaluate the client's mental status for competence to refuse treatment.

A 35-year-old female client with cancer refuses to allow the nurse to insert an IV for a scheduled chemotherapy treatment, and states that she is ready to go home to die. What intervention should the nurse initiate? Review the client's medical record for an advance directive. Determine if a do-not-resuscitate prescription has been obtained. Document that the client is being discharged against medical advice. Evaluate the client's mental status for competence to refuse treatment.

Evaluate the client's mental status for competence to refuse treatment.

A 35-year-old female client with cancer refuses to allow the nurse to insert an IV for a scheduled chemotherapy treatment, and states that she is ready to go home to die. What intervention should the nurse initiate? Review the client's medical record for an advance directive. Determine if a do-not-resuscitate prescription has been obtained. Document that the client is being discharged against medical advice. Evaluate the client's mental status for competence to refuse treatment.

Evaluate the client's mental status for competence to refuse treatment. Rationale Competent clients have the right to refuse treatment, so the nurse should first ensure that the client is competent (D). (A and C) are not necessary for a competent client to refuse treatment. The nurse cannot document (C) until the healthcare provider is notified of the client's wishes and a discharge prescription is obtained.

A nurse is becoming increasingly frustrated by the family members' efforts to participate in the care of a hospitalized client. What action should the nurse implement to cope with these feelings of frustration?

Examine one's own culturally based values, beliefs, attitudes & practices. Rationale: Cultural sensitivity begins with examining one's own cultural values.

A nurse is becoming increasingly frustrated by the family members' efforts to participate in the care of a hospitalized client. What action should the nurse implement to cope with these feelings of frustration?

Examine one's own culturally based values, beliefs, attitudes, and practices.

A nurse is becoming increasingly frustrated by the family members' efforts to participate in the care of a hospitalized client. What action should the nurse implement to cope with these feelings of frustration? Suggest that other cultural practices be substituted by the family members. Examine one's own culturally based values, beliefs, attitudes, and practices. Explain to the family that multiple visitors are exhausting to the client. Allow the situation to continue until a family member's action may harm the client.

Examine one's own culturally based values, beliefs, attitudes, and practices. Rationale Acknowledging a client's beliefs and customs related to sickness and health care are valuable components in the plan of care that prevents conflict between the goals of nursing and the client's cultural practices. Cultural sensitivity begins with examining one's own cultural values to compare, recognize, and acknowledge cultural bias.

What activity should the nurse use in the evaluation phase of the nursing process?

Examine the effectiveness of nursing interventions toward meeting client outcomes

What activity should the nurse use in the evaluation phase of the nursing process?

Examine the effectiveness of nursing interventions toward meeting client outcomes.

What activity should the nurse use in the evaluation phase of the nursing process? Ask a client to evaluate the nursing care provided. Document the nursing care plan in the progress notes. Determine whether a client's health problems have been alleviated. Examine the effectiveness of nursing interventions toward meeting client outcomes.

Examine the effectiveness of nursing interventions toward meeting client outcomes.

Which assessment data provides the most accurate determination of proper placement of a nasogastric tube? Aspirating gastric contents to assure a pH value of 4 or less. Hearing air pass in the stomach after injecting air into the tubing. Examining a chest x-ray obtained after the tubing was inserted. Checking the remaining length of tubing to ensure that the correct length was inserted.

Examining a chest x-ray obtained after the tubing was inserted.

A nurse observes a student nurse taking a copy of a client's medication administration record. What response should the nurse provide first?

Explain that the records are hospital property & may not be removed.

A nurse observes a student nurse taking a copy of a client's medication administration record. When questioned, the student states, "Another student is scheduled to administer medications for this client tomorrow, so I am going to make a copy to help my friend prepare for tomorrow's clinical." What response should the nurse provide first?

Explain that the records are hospital property and may not be removed.

A nurse observes a student nurse taking a copy of a client's medication administration record. When questioned, the student states, "Another student is scheduled to administer medications for this client tomorrow, so I am going to make a copy to help my friend prepare for tomorrow's clinical." What response should the nurse provide first? Ask the nursing supervisor to meet with the students. Notify the student's clinical instructor of the situation. Ask the student if permission was obtained from the client. Explain that the records are hospital property and may not be removed.

Explain that the records are hospital property and may not be removed.

A nurse observes a student nurse taking a copy of a client's medication administration record. When questioned, the student states, "Another student is scheduled to administer medications for this client tomorrow, so I am going to make a copy to help my friend prepare for tomorrow's clinical." What response should the nurse provide first? Ask the nursing supervisor to meet with the students. Notify the student's clinical instructor of the situation. Ask the student if permission was obtained from the client. Explain that the records are hospital property and may not be removed.

Explain that the records are hospital property and may not be removed. Rationale The nurse should deal with the issue immediately and explain that a client's records are the property of the hospital and cannot be removed (D), even with the client's permission (C). Next, the clinical instructor should be notified (B) so that all students can be educated regarding copying and removing clinical records from the healthcare agency. The nursing supervisor (A) should also be alerted to ensure appropriate supervision of students as well as protection of client information.

The nurse notices that the mother a 9-year-old Vietnamese child always looks at the floor when she talks to the nurse. What action should the nurse take? A. Talk directly to the child instead of the mother. B. Continue asking the mother questions about the child. C. Ask another nurse to interview the mother now. D. Tell the mother politely to look at you when answering.

Eye contact is a culturally-influenced form of non-verbal communication. In some non-Western cultures, such as the Vietnamese culture, a client or family member may avoid eye contact as a form of respect, so the nurse should continue to ask the mother questions about the child (B). (A, C, and D) are not indicated. Correct Answer: B

When evaluating a client's plan of care, the nurse determines that a desired outcome was not achieved. Which action will the nurse implement first? A. Establish a new nursing diagnosis. B. Note which actions were not implemented. C. Add additional nursing orders to the plan. D. Collaborate with the healthcare provider to make changes.

First, the nurse reviews which actions in the original plan were not implemented (B) in order to determine why the original plan did not produce the desired outcome. Appropriate revisions can then be made, which may include revising the expected outcome, or identifying a new nursing diagnosis (A). (C) may be needed if the nursing actions were unsuccessful, or were unable to be implemented. (D) other members of the healthcare team may be necessary to collaborate changes once the nurse determines why the original plan did not produce the desired outcome. Correct Answer: B

The home health nurse visits an elderly client who lives at home with her husband. She experiences frequent episodes of diarrhea & bowel incontinence. Which problem, for which the client is at risk, has the greatest priority when planning the client's care?

Fluid volume imbalance

The home health nurse visits an elderly client who lives at home with her husband. The client is experiencing frequent episodes of diarrhea and bowel incontinence. Which problem, for which the client is at risk, has the greatest priority when planning the client's care?

Fluid volume imbalance.

When preparing to administer IV med through central venous catheter, nurse aspirated blood return in one of the lumens of the triple lumen catheter. Which action should the nurse implement?

Flush the lumen with the saline solution & administer the medication through the lumen

When preparing to administer an intravenous medication through a central venous catheter, the nurse aspirates a blood return in one of the lumens of the triple lumen catheter. Which action should the nurse implement? Flush the lumen with the saline solution and administer the medication through the lumen. Determine if a PRN prescription for a thrombolytic agent is listed on the medication record. Clamp the lumen and obtain a syringe of a dilute heparin solution to flush through the tubing. Withdraw the aspirated blood into the syringe and use a new syringe to administer the medication.

Flush the lumen with the saline solution and administer the medication through the lumen.

When preparing to administer an intravenous medication through a central venous catheter, the nurse aspirates a blood return in one of the lumens of the triple lumen catheter. Which action should the nurse implement? Flush the lumen with the saline solution and administer the medication through the lumen. Determine if a PRN prescription for a thrombolytic agent is listed on the medication record. Clamp the lumen and obtain a syringe of a dilute heparin solution to flush through the tubing. Withdraw the aspirated blood into the syringe and use a new syringe to administer the medication.

Flush the lumen with the saline solution and administer the medication through the lumen. Rationale Aspiration of a blood return in the lumen of a central venous catheter indicates that the catheter is in place and the medication can be administered. The nurse should flush the tubing with the saline solution, administer the medication (A), then flush the lumen with saline again. (B and C) are not necessary. The aspirated blood can be flushed back through the closed system into the client's bloodstream, but does not need to be withdrawn (D).

A 73y/o Hispanic client is seen with a history of protein malnutrition. What information should the nurse obtain first?

Foods & liquids consumed during past 24 hrs Rationale: Client's dietary habits should be determined first through dietary recall before suggesting protein sources or supplements as options in client's diet.

A 73-year-old Hispanic client is seen at the community health clinic with a history of protein malnutrition. What information should the nurse obtain first?

Foods and liquids consumed during the past 24 hours.

A client with chronic renal failure selects a scrambled egg for his breakfast. What action should the nurse take? A. Commend the client for selecting a high biologic value protein. B. Remind the client that protein in the diet should be avoided. C. Suggest that the client also select orange juice, to promote absorption. D. Encourage the client to attend classes on dietary management of CRF.

Foods such as eggs and milk (A) are high biologic proteins which are allowed because they are complete proteins and supply the essential amino acids that are necessary for growth and cell repair. Although a low-protein diet is followed (B), some protein is essential. Orange juice is rich in potassium, and should not be encouraged (C). The client has made a good diet choice, so (D) is not necessary. Correct Answer: A

The nurse is completing a mental assessment for a client who is demonstrating slow thought processes, personality changes, and emotional lability. Which area of the brain controls these neuro-cognitive functions? Thalamus. Hypothalamus. Frontal lobe. Parietal lobe.

Frontal lobe.

Middle-aged woman enjoys being a teacher & mentor. Feels that she should pass down her legacy of knowledge & skills to the younger generation. According to Erikson, she is involved in what developmental stage?

Generativity

A middle-aged woman who enjoys being a teacher and mentor feels that she should pass down her legacy of knowledge and skills to the younger generation. According to Erikson, she is involved in what developmental stage? Generativity. Ego integrity. Identification. Valuing wisdom.

Generativity.

A client is in the radiology department at 0900 when the prescription levofloxacin (Levaquin) 500 mg IV every 24 hours is scheduled to be administered. The client returns to the unit at 1300. What is the best intervention for the nurse to implement? Contact the healthcare provider and complete a medication variance form. Administer the Levaquin at 1300 and resume the 0900 schedule in the morning. Notify the charge nurse and complete an incident report to explain the missed dose. Give the missed dose at 1300 and change the schedule to administer daily at 1300.

Give the missed dose at 1300 and change the schedule to administer daily at 1300.

Which action is most important for the nurse to implement when donning sterile gloves? A. Maintain thumb at a ninety degree angle. B. Hold hands with fingers down while gloving. C. Keep gloved hands above the elbows. D. Put the glove on the dominant hand first.

Gloved hands held below waist level are considered unsterile (C). (A and B) are not essential to maintaining asepsis. While it may be helpful to put the glove on the dominant hand first, it is not necessary to ensure asepsis (D). Correct Answer: C

The nurse overhears the HCP explaining to the client that the tumor removed was non-malignant & that the client will be fine. Nurse has read in the pathology report that the tumor was malignant & that there is extensive metastasis. Who should the nurse consult with first regarding the situation?

HCP

A resident in a skilled nursing facility for short-term rehabilitation after a hip replacement tells the nurse, "I don't want any more blood taken for those useless tests." Which narrative documentation should the nurse enter in the client's medical record? Healthcare provider notified of failure to collect specimens for prescribed blood studies. Blood specimens not collected because client no longer wants blood tests performed. Healthcare provider notified of client's refusal to have blood specimens collected for testing. Client irritable, uncooperative, and refuses to have blood collected. Healthcare provider notified.

Healthcare provider notified of client's refusal to have blood specimens collected for testing.

The nurse overhears the healthcare provider explaining to the client that the tumor removed was non-malignant and that the client will be fine. However, the nurse has read in the pathology report that the tumor was malignant and that there is extensive metastasis. Who should the nurse consult with first regarding the situation?

Healthcare provider.

The nurse overhears the healthcare provider explaining to the client that the tumor removed was non-malignant and that the client will be fine. However, the nurse has read in the pathology report that the tumor was malignant and that there is extensive metastasis. Who should the nurse consult with first regarding the situation? Healthcare provider. Client's family. Case manager. Chief of staff.

Healthcare provider.

A female client informs the nurse that she uses herbal therapies to supplement her diet and manage common ailments. What information should the nurse offer the client about general use of herbal supplements?

Herbs should be obtained from manufacturers with a history of quality control of their supplements.

A female client informs the nurse that she uses herbal therapies to supplement her diet and manage common ailments. What information should the nurse offer the client about general use of herbal supplements? Most herbs are toxic or carcinogenic and should be used only when proven effective. There is no evidence that herbs are safe or effective as compared to conventional supplements in maintaining health. Herbs should be obtained from manufacturers with a history of quality control of their supplements. Herbal therapies may mask the symptoms of serious disease, so frequent medical evaluation is required during use.

Herbs should be obtained from manufacturers with a history of quality control of their supplements. Rationale The current availability of many herbal supplements lacks federal regulation, research, control and standardization in the manufacture of its purity and dose. Manufacturers that provide evidence of quality control (C), such as labeling that contains scientific generic name, name and address of the manufacturer, batch or lot number, date of manufacture, and expiration date, is the best information to provide. (A, B, and D) are misleading.

Client informs the nurse that she uses herbal therapies. What information should the nurse offer about general use of herbal supplements?

Herbs should be obtained from manufacturers with history of quality control of their supplements

25. After the catheter is removed, Susan calls the nurse with a complaint of feeling dizzy. The nurse finds that the client's vital signs are BP 82/56 and HR 62 bpm. When the nurse inspects the catheter site, the nurse observes a large area of blood on the sheets, and active bleeding coming from under the groin-site dressing. What is the top priority?

Holding pressure over the site and calling for help

HCP is performing a sterile procedure at client's bedside. Nurse observes HCP break sterility. What is the best action for the nurse to implement?

Identify the break in surgical asepsis & provide another set of sterile supplies

A healthcare provider is performing a sterile procedure at a client's bedside. Near the end of the procedure, the nurse observes the healthcare provider contaminate a sterile glove and the sterile field. What is the best action for the nurse to implement?

Identify the break in surgical asepsis and provide another set of sterile supplies.

A healthcare provider is performing a sterile procedure at a client's bedside. Near the end of the procedure, the nurse observes the healthcare provider contaminate a sterile glove and the sterile field. What is the best action for the nurse to implement? Report the healthcare provider for the violation in aseptic technique. Allow the completion of the procedure. Ask if the glove and sterile field are contaminated. Identify the break in surgical asepsis and provide another set of sterile supplies.

Identify the break in surgical asepsis and provide another set of sterile supplies.

At the beginning of the shift, the nurse assesses a client who is admitted from the post-anesthesia care unit (PACU). When should the nurse document the client's findings? At the beginning, middle, and end of the shift. After client priorities are identified for the development of the nursing care plan. At the end of the shift so full attention can be given to the client's needs. Immediately after the assessments are completed.

Immediately after the assessments are completed.

When caring for an immobile client, what nursing diagnosis has the highest priority?

Impaired gas exchange

When caring for an immobile client, what nursing diagnosis has the highest priority?

Impaired gas exchange.

When caring for an immobile client, what nursing diagnosis has the highest priority? Risk for fluid volume deficit. Impaired gas exchange. Risk for impaired skin integrity. Altered tissue perfusion.

Impaired gas exchange.

Determine if the expected outcomes of care were achieved.

In evaluating client care, which action should the nurse take first?

Notify family members of the client's condition.

In providing care for a terminally ill resident of a long-term care facility, the nurse determines that the resident is exhibiting signs of impending death and has a "do not resuscitate" or DNR status. What intervention should the nurse implement first?

Male client with venous incompetence stands up and his BP subsequently drops. Which finding should the nurse identify as a compensatory response?

Increase in pulse rate

A male client with venous incompetence stands up and his blood pressure subsequently drops. Which finding should the nurse identify as a compensatory response? Bradycardia. Increase in pulse rate. Peripheral vasodilation. Increase in cardiac output.

Increase in pulse rate. Rationale When postural hypotension occurs, the body attempts to restore arterial pressure by stimulating the baro-receptors to increase the heart rate (B), not decrease it (A). Peripheral vasoconstriction, not dilation (C), of the veins and arterioles occurs with venous incompetence through the baro-receptor reflex. A decrease in cardiac output, not an increase (D), occurs when orthostatic hypotension occurs.

Which statement is an example of a correctly written nursing diagnosis statement?

Ineffective coping related to response to positive biopsy test results Rationale: "Diagnostic label" followed by "related to" the cause, which should direct the nurse to the appropriate interventions. Should not include medical diagnosis. Should not focus on client's response.

Client in hospice care develops audible gurgling sounds on inspiration. Which nursing action has highest priority?

Inform the family that death is imminent

A client in hospice care develops audible gurgling sounds on inspiration. Which nursing action has the highest priority?

Inform the family that death is imminent.

A client in hospice care develops audible gurgling sounds on inspiration. Which nursing action has the highest priority? Ensure cultural customs are observed. Increase oxygen flow to 4L/minute. Auscultate bilateral lung fields. Inform the family that death is imminent.

Inform the family that death is imminent.

An African-American grandmother tells the nurse that her 4-year-old grandson is suffering with "miseries." Based on this statement, which focused assessment should the nurse conduct? Inquire about the source and type of pain. Examine the nose for congestion and discharge. Take vital signs for temperature elevation. Explore the abdominal area for distension.

Inquire about the source and type of pain.

Three days following surgery, a male client observes his colostomy for the first time. He becomes quite upset and tells the nurse that it is much bigger than he expected. What is the best response by the nurse? Reassure the client that he will become accustomed to the stoma appearance in time. Instruct the client that the stoma will become smaller when the initial swelling diminishes. Offer to contact a member of the local ostomy support group to help him with his concerns. Encourage the client to handle the stoma equipment to gain confidence with the procedure.

Instruct the client that the stoma will become smaller when the initial swelling diminishes.

While preparing to insert a rectal suppository into a male adult client, the nurse observes the client holding his breath while bearing down. What action should the nurse implement?

Instruct the client to take slow deep breaths & stop bearing down

While preparing to insert a rectal suppository in a male adult client, the nurse observes that the client is holding his breath while bearing down. What action should the nurse implement?

Instruct the client to take slow deep breaths and stop bearing down.

27. Which lab result requires immediate action by the nurse?

International normalized ration (INR) 5

The nurse assigns a UAP to obtain vital signs from a very anxious client. What instructions should the nurse give the UAP? A. Remain calm with the client and record abnormal results in the chart. B. Notify the medication nurse immediately if the pulse or blood pressure is low. C. Report the results of the vital signs to the nurse. D. Reassure the client that the vital signs are normal.

Interpretation of vital signs is the responsibility of the nurse, so the UAP should report vital sign measurements to the nurse (C). (A, B, and D) require the UAP to interpret the vital signs, which is beyond the scope of the UAP's authority. Correct Answer: C

A male client tells the nurse that he does not know where he is or what year it is. What data should the nurse document that is most accurate? Demonstrates loss of remote memory. Exhibits expressive dysphasia. Has a diminished attention span. Is disoriented to place and time.

Is disoriented to place and time.

6. Which data places the client at highest risk for falls?

Knee pain

The nurse is instructing a client with high cholesterol about diet and life style modification. What comment from the client indicates that the teaching has been effective? A. If I exercise at least two times weekly for one hour, I will lower my cholesterol. B. I need to avoid eating proteins, including red meat. C. I will limit my intake of beef to 4 ounces per week. D. My blood level of low density lipoproteins needs to increase.

Limiting saturated fat from animal food sources to no more than 4 ounces per week (C) is an important diet modification for lowering cholesterol. To be effective in reducing cholesterol, the client should exercise 30 minutes per day, or at least 4 to 6 times per week (A). Red meat and all proteins do not need to be eliminated (B) to lower cholesterol, but should be restricted to lean cuts of red meat and smaller portions (2-ounce servings). The low density lipoproteins (D) need to decrease rather than increase. Correct Answer: C

During the daily nursing assessment, a client begins to cry and states that the majority of family and friends have stopped calling and visiting. What action should the nurse take? Listen and show interest as the client expresses these feelings. Reinforce that this behavior means they were not true friends. Ask the healthcare provider for a psychiatric consult. Continue with the assessment and tell the client not to worry.

Listen and show interest as the client expresses these feelings.

When teaching female client to perform intermittent self-catheterization, the nurse should ensure client's ability to perform which action?

Locate the perineum

When teaching a female client to perform intermittent self-catheterization, the nurse should ensure the client's ability to perform which action?

Locate the perineum.

When teaching a female client to perform intermittent self-catheterization, the nurse should ensure the client's ability to perform which action? Locate the perineum. Transfer to a commode. Attach the catheter to a drainage bag. Manipulate a syringe to inflate the balloon.

Locate the perineum.

Examination of a client complaining of itching on his right arm reveals a rash made up of multiple flat areas of redness ranging from pinpoint to 0.5 cm in diameter. How should the nurse record this finding? A. Multiple vesicular areas surrounded by redness, ranging in size from 1 mm to 0.5 cm. B. Localized red rash comprised of flat areas, pinpoint to 0.5 cm in diameter. C. Several areas of red, papular lesions from pinpoint to 0.5 cm in size. D. Localized petechial areas, ranging in size from pinpoint to 0.5 cm in diameter.

Macules are localized flat skin discolorations less than 1 cm in diameter. However, when recording such a finding the nurse should describe the appearance (B) rather than simply naming the condition. (A) identifies vesicles -- fluid filled blisters -- an incorrect description given the symptoms listed. (C) identifies papules -- solid elevated lesions, again not correctly identifying the symptoms. (D) identifies petechiae -- pinpoint red to purple skin discolorations that do not itch, again an incorrect identification. Correct Answer: B

The unlicensed assistive personnel (UAP) working on a chronic neuro unit asks the nurse to help determine the safest way to transfer an older client with left-sided weakness from the bed to the chair. Which method describes the correct transfer procedure for this client? Place the chair at a right angle to the bed on the client's left side before moving. Assist the client to a standing position, then place the right hand on the armrest. Have the client place the left foot next to the chair and pivot to the left before sitting. Move the chair parallel to the right side of the bed, and stand the client on the right foot.

Move the chair parallel to the right side of the bed, and stand the client on the right foot.

When evaluating a client's plan of care, the nurse determines that a desired outcome was not achieved. Which action should the nurse implement first? Establish a new nursing diagnosis. Note which actions were not implemented. Add additional nursing orders to the plan. Collaborate with the healthcare provider to make changes.

Note which actions were not implemented.

Terminally ill patient in LTC exhibits signs of impending death & has a DNR status. What intervention should the nurse implement first?

Notify family members of the client's condition

In providing care for a terminally ill resident of a long-term care facility, the nurse determines that the resident is exhibiting signs of impending death and has a "do not resuscitate" or DNR status. What intervention should the nurse implement first?

Notify family members of the client's condition.

In providing care for a terminally ill resident of a long-term care facility, the nurse determines that the resident is exhibiting signs of impending death and has a "do not resuscitate" or DNR status. What intervention should the nurse implement first? Request hospice care for the client. Report the client's acuity level to the nursing supervisor. Notify family members of the client's condition. Inform the chaplain that the client's death is imminent.

Notify family members of the client's condition. Rationale The nurse's first priority is to notify the family of the resident's impending death (C). The family may request that hospice care is initiated (A). Reporting the client's acuity level (B) does not have the priority of informing the family of the client's condition. Once the family is contacted, the nurse can also contact the chaplain (D).

After a client has been premedicated for surgery with an opioid analgesic, the nurse discovers that the operative permit has not been signed. What action should the nurse implement?

Notify the surgeon that the consent form has not been signed Rationale: Once a client has been premedicated for surgery with any type of sedative, legal informed consent is not possible, so the nurse must notify the surgeon. Remaining options are not legally viable options for ensuring informed consent.

The nurse observes that a male client has removed the covering from an ice pack applied to his knee. What action should the nurse take first? Observe the appearance of the skin under the ice pack. Instruct the client regarding the need for the covering. Reapply the covering after filling with fresh ice. Ask the client how long the ice was applied to the skin.

Observe the appearance of the skin under the ice pack.

Which nursing intervention is most beneficial in reducing the risk of urosepsis in a hospitalized client with an indwelling urinary catheter?

Obtain a prescription for removal of the catheter as soon as possible.

Which nursing intervention is most beneficial in reducing risk of urosepsis in a hospitalized client with an indwelling urinary catheter?

Obtain prescription for removal of catheter ASAP

29. Which member of the healthcare team is most appropriate to help the client learn how to cook while using a walker?

Occupational therapist

Provide warm prune juice before the client goes to bed at night.

On the third postoperative day following thoracic surgery, a client reports feeling constipated. Which intervention should the nurse implement to promote bowel elimination?

71.After a client has been premedicated for surgery with an opioid analgesic, the nurse discovers that the operative permit has not been signed. What action should the nurse implement? A. Notify the surgeon that the consent form has not been signed. B. Read the consent form to the client before witnessing the client's signature. C. Determine if the client's spouse is willing to sign the consent form. D. Administer an opioid antagonist prior to obtaining the client's signature.

Once a client has been premedicated for surgery with any type of sedative, legal informed consent is not possible, so the nurse must notify the surgeon (A). (B, C, and D) are not legally viable options for ensuring informed consent. Correct Answer: A

The nurse removes the dressing on a client's heel that is cover a pressure sore 1" in diameter & finds that there is straw-colored drainage seeping from the wound. What description of this finding should the nurse include in the client's record?

One-inch pressure sore draining serous fluid

The nurse removes the dressing on a client's heel that is covering a pressure sore one-inch in diameter and finds that there is straw-colored drainage seeping from the wound. What description of this finding should the nurse include in the client's record?

One-inch pressure sore draining serous fluid.

A client's infusion of normal saline infiltrated earlier today, and approximately 500 ml of saline infused into the subcutaneous tissue. The client is now complaining of excruciating arm pain and demanding "stronger pain medications." What initial action is most important for the nurse to take? A. Ask about any past history of drug abuse or addiction. B. Measure the pulse volume and capillary refill distal to the infiltration. C. Compress the infiltrated tissue to measure the degree of edema. D. Evaluate the extent of ecchymosis over the forearm area.

Pain and diminished pulse volume (B) are signs of compartment syndrome, which can progress to complete loss of the peripheral pulse in the extremity. Compartment syndrome occurs when external pressure (usually from a cast), or internal pressure (usually from subcutaneous infused fluid), exceeds capillary perfusion pressure resulting in decreased blood flow to the extremity. (A) should not be pursued until physical causes of the pain are ruled out. (C) is of less priority than determining the effects of the edema on circulation and nerve function. Further assessment of the client's ecchymosis can be delayed until the signs of edema and compression that suggest compartment syndrome have been examined (D). Correct Answer: B

Older client who is able to stand but not ambulate receives a prescription to be mobilized into a chair as tolerated during each day. What is the best action for the nurse to implement when assisting the client from the bed to the chair?

Place a transfer belt around the client, assist to stand & pivot to a chair that is placed at a right angle to bed

An older client who is able to stand but not to ambulate receives a prescription to be mobilized into a chair as tolerated during each day. What is the best action for the nurse to implement when assisting the client from the bed to the chair?

Place a transfer belt around the client, assist to stand, and pivot to a chair that is placed at a right angle to the bed.

23. Which action should the nurse take before leaving the client's room to call the surgeon?

Place the client on strict bedrest and elevate the left leg on a pillow

How should the nurse handle linens that are soiled with incontinent feces?

Place the soiled linens in a pillow case & deposit them in the dirty linen hamper

How should the nurse handle linens that are soiled with incontinent feces?

Place the soiled linens in a pillow case and deposit them in the dirty linen hamper.

What action should the nurse implement to prevent the formation of a sacral ulcer for a client who is immobile?

Position prone with a small pillow below the diaphragm

What action should the nurse implement to prevent the formation of a sacral ulcer for a client who is immobile?

Position prone with a small pillow below the diaphragm.

What action should the nurse implement to prevent the formation of a sacral ulcer for a client who is immobile? Maintain in a lateral position using protective wrist and vest devices. Position prone with a small pillow below the diaphragm. Raise the head and knee gatch when lying in a supine position. Transfer into a wheelchair close to the nurse's station for observation.

Position prone with a small pillow below the diaphragm.

What action should the nurse implement to prevent the formation of a sacral ulcer for a client who is immobile? Maintain in a lateral position using protective wrist and vest devices. Position prone with a small pillow below the diaphragm. Raise the head and knee gatch when lying in a supine position. Transfer into a wheelchair close to the nurse's station for observation.

Position prone with a small pillow below the diaphragm. Rationale The prone position (B) using a small pillow below the diaphragm maintains alignment and provides the best pressure relief over the sacral bony prominence. Using protective (restraining) devices (A) is not indicated. Raising the head and bed gatch (C) may reduce shearing forces due to sliding down in bed, but it interferes with venous return from the legs and places pressure on the sacrum, predisposing to ulcer formation. Sitting in a wheelchair (D) places the body weight over the ischial tuberosities and predisposes to a potential pressure point.

Three days following surgery, a male client observes his colostomy for the first time. He becomes quite upset and tells the nurse that it is much bigger than he expected. What is the best response by the nurse? A. Reassure the client that he will become accustomed to the stoma appearance in time. B. Instruct the client that the stoma will become smaller when the initial swelling diminishes. C. Offer to contact a member of the local ostomy support group to help him with his concerns. D. Encourage the client to handle the stoma equipment to gain confidence with the procedure.

Postoperative swelling causes enlargement of the stoma. The nurse can teach the client that the stoma will become smaller when the swelling is diminished (B). This will help reduce the client's anxiety and promote acceptance of the colostomy. (A) does not provide helpful teaching or support. (C) is a useful action, and may be taken after the nurse provides pertinent teaching. The client is not yet demonstrating readiness to learn colostomy care (D). Correct Answer: B

The nurse is preparing a male client who has an indwelling catheter and an IV infusion to ambulate from the bed to a chair for the first time following abdominal surgery. What action(s) should the nurse implement prior to assisting the client to the chair? (Select all that apply.)

Pre-medicate the client with an analgesic. Inform the client of the plan for moving to the chair. Ask the client to push the IV pole to the chair. Assess the client's blood pressure.

On the third postoperative day following thoracic surgery, a client reports feeling constipated. Which intervention should the nurse implement to promote bowel elimination?

Provide warm prune juice before the client goes to bed at night

On the third postoperative day following thoracic surgery, a client reports feeling constipated. Which intervention should the nurse implement to promote bowel elimination?

Provide warm prune juice before the client goes to bed at night.

On the third postoperative day following thoracic surgery, a client reports feeling constipated. Which intervention should the nurse implement to promote bowel elimination? Remind the client to turn every two hours while lying in bed. Provide warm prune juice before the client goes to bed at night. Teach the client to splint the incision while walking to the bathroom. Administer an analgesic before the client attempts to defecate.

Provide warm prune juice before the client goes to bed at night. Rationale Prune juice is a natural laxative that stimulates peristalsis, and warming the prune juice (B) facilitates peristalsis. (A) is also helpful in promoting peristalsis but is less likely to relieve the client's constipation. (C) reduces discomfort during ambulation, but will not help relieve the client's constipation. Defecation is not painful following most surgeries, and many analgesics used postoperatively cause constipation, so (D) is contraindicated.

The nurse is digitally removing a fecal impaction for a client. The nurse should stop the procedure & take corrective action if which client reaction is noted?

Pulse rate decreases from 78 to 52 beats/min

The nurse is digitally removing a fecal impaction for a client. The nurse should stop the procedure and take corrective action if which client reaction is noted?

Pulse rate decreases from 78 to 52 beats/min.

The nurse is digitally removing a fecal impaction for a client. The nurse should stop the procedure and take corrective action if which client reaction is noted? Temperature increases from 98.8 to 99.0 F. Pulse rate decreases from 78 to 52 beats/min. Respiratory rate increases from 16 to 24 breaths/min. Blood pressure increases from 110/84 to 118/88 mm/Hg.

Pulse rate decreases from 78 to 52 beats/min.

What action is most important for the nurse to implement when placing a client in the Sim's position?

Raise the bed to a waist-high working level.

What action is most important for the nurse to implement when placing a client in the Sim's position? Raise the bed to a waist-high working level. Elevate the head of the bed 45 degrees. Place a pillow behind the client's back. Bring the client to one edge of the bed.

Raise the bed to a waist-high working level.

What is most important for the nurse to implement when placing a client in the Sim's position?

Raise the bed to a wasit-high working level

An older client who is a resident in a long term care facility has been bedridden for a week. Which finding should the nurse identify as a client risk factor for pressure ulcers? Generalized dry skin. Localized dry skin on lower extremities. Red flush over entire skin surface. Rashes in the axillary, groin, and skin fold regions.

Rashes in the axillary, groin, and skin fold regions.

The nurse is performing nasotracheal suctioning. After suctioning the client's trachea for fifteen seconds, large amounts of thick yellow secretions return. What action should the nurse implement next? Encourage the client to cough to help loosen secretions. Advise the client to increase the intake of oral fluids. Rotate the suction catheter to obtain any remaining secretions. Re-oxygenate the client before attempting to suction again.

Re-oxygenate the client before attempting to suction again.

The nurse observes an unlicensed assistive personnel (UAP) checking a client's blood pressure with a cuff that is too small, but the blood pressure reading obtained is within the client's usual range. What action is most important for the nurse to implement? Tell the UAP to use a larger cuff at the next scheduled assessment. Reassess the client's blood pressure using a larger cuff. Have the unit educator review this procedure with the UAPs. Teach the UAP the correct technique for assessing blood pressure.

Reassess the client's blood pressure using a larger cuff.

22. Which finding is of most concern?

Red, warm, and edematous area of the left calf

The charge nurse assigns a nursing procedure to a new staff nurse who has not previously performed the procedure. What action is most important for the new staff nurse to take?

Refuse to perform the task that is beyond the nurse's experience Rationale: According to states' nurse practice acts, it is the responsibility of the nurse to function within the scope of competency.

The charge nurse assigns a nursing procedure to a new staff nurse who has not previously performed the procedure. What action is most important for the new staff nurse to take?

Refuse to perform the task that is beyond the nurse's experience.

The charge nurse assigns a nursing procedure to a new staff nurse who has not previously performed the procedure. What action is most important for the new staff nurse to take? Review the steps in the procedure manual. Ask another nurse to assist while implementing the procedure. Follow the agency's policy and procedure. Refuse to perform the task that is beyond the nurse's experience.

Refuse to perform the task that is beyond the nurse's experience.

The charge nurse assigns a nursing procedure to a new staff nurse who has not previously performed the procedure. What action is most important for the new staff nurse to take? Review the steps in the procedure manual. Ask another nurse to assist while implementing the procedure. Follow the agency's policy and procedure. Refuse to perform the task that is beyond the nurse's experience.

Refuse to perform the task that is beyond the nurse's experience. Rationale According to states' nurse practice acts, it is the responsibility of the nurse to function within the scope of competency (D), and in this case safe nursing practice constitutes refusal to perform the procedure because of a lack of experience. Although state mandates, agency policies, and continued education and experience identify tasks that are within the scope of nursing practice, nurses should first refuse to perform tasks that are beyond their proficiency, and then pursue opportunities to enhance their

In developing a plan of care for a client with dementia, the nurse should remember that confusion in the elderly A. is to be expected, and progresses with age. B. often follows relocation to new surroundings. C. is a result of irreversible brain pathology. D. can be prevented with adequate sleep.

Relocation (B) often results in confusion among elderly clients--moving is stressful for anyone. (A) is a stereotypical judgment. Stress in the elderly often manifests itself as confusion, so (C) is wrong. Adequate sleep is not a prevention (D) for confusion. Correct Answer: B

4. When the nurse is teaching Susan about medications for osteoporosis, which instruction is most important, knowing that Susan takes alendronate (Fosamax), a biphosphonate?

Remain upright (sitting or standing) for at least 30 minutes after taking this medication

Nurse is administering intermittent infusion of abx to a client whose IV access is an AC saline lock. After nurse opens roller clamp on the IV tubing, the alarm on the pump indicates an obstruction. What actions should the nurse take first?

Reposition the client's arm

The nurse is administering an intermittent infusion of an antibiotic to a client whose intravenous (IV) access is an antecubital saline lock. After the nurse opens the roller clamp on the IV tubing, the alarm on the infusion pump indicates an obstruction. What action should the nurse take first?

Reposition the client's arm.

The nurse is administering an intermittent infusion of an antibiotic to a client whose intravenous (IV) access is an antecubital saline lock. After the nurse opens the roller clamp on the IV tubing, the alarm on the infusion pump indicates an obstruction. What action should the nurse take first? Check for a blood return. Reposition the client's arm. Remove the IV site dressing. Flush the lock with saline.

Reposition the client's arm.

Which client assessment data is most important for the nurse to consider before ambulating post-op client?

Respiratory rate Rationale: mobilization & ambulation increase oxygen use

Which client assessment data is most important for the nurse to consider before ambulating a postoperative client?

Respiratory rate.

Nurse notes that client consistently coughs when eating & drinking. Which nursing diagnosis is most important for the nurse to include in patient's care plan?

Risk for aspiration

Client who has breast cancer with metastasis to the liver & spine is admitted with constant, severe pain despite around-the-clock use of oxycodone & amitriptyline for pain control at home. During the admission assessment, which information is most important for the nurse to obtain?

Sensory pattern, area, intensity & nature of pain

A female client who has breast cancer with metastasis to the liver and spine is admitted with constant, severe pain despite around-the-clock use of oxycodone (Percodan) and amitriptyline (Elavil) for pain control at home. During the admission assessment, which information is most important for the nurse to obtain?

Sensory pattern, area, intensity, and nature of the pain.

A client with chronic renal disease is admitted to the hospital for evaluation prior to a surgical procedure. Which laboratory test indicates client's protein status for the longest length of time?

Serum albumin Rationale: Serum albumin has a long half-life.

A client with chronic renal disease is admitted to the hospital for evaluation prior to a surgical procedure. Which laboratory test indicates the client's protein status for the longest length of time?

Serum albumin.

A client with chronic renal disease is admitted to the hospital for evaluation prior to a surgical procedure. Which laboratory test indicates the client's protein status for the longest length of time? Transferrin. Prealbumin. Serum albumin. Urine urea nitrogen.

Serum albumin.

A client with chronic renal disease is admitted to the hospital for evaluation prior to a surgical procedure. Which laboratory test indicates the client's protein status for the longest length of time? Transferrin. Prealbumin. Serum albumin. Urine urea nitrogen.

Serum albumin. Rationale Serum albumin has a long half-life and is the best long-term indicator of the body's entry into a catabolic state following protein depletion from malnutrition or stress of chronic illness (C). While (A) is a good indicator of iron-binding capacity in a healthy adult, it is an unreliable measure in the client with a chronic illness. (B) has a short half-life, and is a sensitive indicator of recent catabolic changes, but it is not as effective as (C) in indicating long-term protein depletion. While (D) is a good indicator of a negative nitrogen balance, it is not as good an indicator of long-term protein catabolism as is (C).

The nurse is evaluating client learning about a low-sodium diet. Selection of which meal would indicate to the nurse that this client understands the dietary restrictions? Tossed salad, low-sodium dressing, bacon and tomato sandwich. New England clam chowder, no-salt crackers, fresh fruit salad. Skim milk, turkey salad, roll, and vanilla ice cream. Macaroni and cheese, diet Coke, a slice of cherry pie.

Skim milk, turkey salad, roll, and vanilla ice cream.

The nurse is evaluating client learning about a low-sodium diet. Selection of which meal would indicate to the nurse that this client understands the dietary restrictions? A. Tossed salad, low-sodium dressing, bacon and tomato sandwich. B. New England clam chowder, no-salt crackers, fresh fruit salad. C. Skim milk, turkey salad, roll, and vanilla ice cream. D. Macaroni and cheese, diet Coke, a slice of cherry pie.

Skim milk, turkey, bread, and ice cream (C), while containing some sodium, are considered low-sodium foods. Bacon (A), canned soups (B), especially those with seafood, hard cheeses, macaroni, and most diet drinks (D) are very high in sodium. Correct Answer: C

39.When assessing a client with a nursing diagnosis of fluid volume deficit, the nurse notes that the client's skin over the sternum "tents" when gently pinched. Which action should the nurse implement? A. Confirm the finding by further assessing the client for jugular vein distention. B. Offer the client high protein snacks between regularly scheduled mealtimes. C. Continue the planned nursing interventions to restore the client's fluid volume. D. Change the plan of care to include a nursing diagnosis of impaired skin integrity.

Skin turgor is assessed by pinching the skin and observing for tenting. This finding confirms the diagnosis of fluid volume deficit, so the nurse should continue interventions to restore the client's fluid volume (C). Jugular vein distention (A) is a sign of fluid volume overload. High protein snacks (B) will not resolve the fluid volume deficit. Changes in the client's skin integrity are not evident (D). Correct Answer: C

Nurse determines client's IV solution is infusing at 250 mL/hr. Prescribed rate is 125 mL/hr. What action should the nurse take first?

Slow the IV infusion to keep vein open (KVO) rate Rationale: Should change flow rate to KVO to prevent further risk of fluid volume overload, then gather additional assessment data, such as when the IV solution was started & appearance of IV site before contacting HCP

The nurses determines a client's IV solution is infusing at 250 ml/hr. The prescribed rate is 125 ml/hr. What action should the nurse take first?

Slow the IV infusion to keep vein open rate.

The nurses determines a client's IV solution is infusing at 250 ml/hr. The prescribed rate is 125 ml/hr. What action should the nurse take first? Determine when the IV solution was started. Slow the IV infusion to keep vein open rate. Assess the IV insertion site for swelling. Report the finding to the healthcare provider.

Slow the IV infusion to keep vein open rate.

The nurses determines a client's IV solution is infusing at 250 ml/hr. The prescribed rate is 125 ml/hr. What action should the nurse take first? Determine when the IV solution was started. Slow the IV infusion to keep vein open rate. Assess the IV insertion site for swelling. Report the finding to the healthcare provider.

Slow the IV infusion to keep vein open rate. Rationale The nurse should first slow the IV flow rate to keep vein open (KVO) rate (B) to prevent further risk of fluid volume overload, then gather additional assessment data, such as when the IV solution was started (A) and the appearance of the IV insertion site (C) before contacting the healthcare provider (D) for further instructions.

A low-sodium, low-protein diet is prescribed for a 45-year-old client with renal insufficiency and hypertension, who gained 3 pounds in the last month. The nurse determines that the client has been noncompliant with the diet, based on which report from the 24-hour dietary recall? (Select all that apply.)

Snack of potato chips, and diet soda. Lunch of tuna fish sandwich, carrot sticks, fresh fruit, and coffee. Breakfast of eggs, bacon, toast, and coffee. Bedtime snack of crackers and milk.

While caring for a child & mother from Cambodia, which action should the nurse implement to accommodate the clients' cultural needs?

Speak initially with the oldest family member to show respect

17. What is the priority intervention?

Stop the blood transfusion and hang a new normal saline infusion

The nurse is performing nasotracheal suctioning. After suctioning the client's trachea for fifteen seconds, large amounts of thick yellow secretions return. What action should the nurse implement next? A. Encourage the client to cough to help loosen secretions. B. Advise the client to increase the intake of oral fluids. C. Rotate the suction catheter to obtain any remaining secretions. D. Re-oxygenate the client before attempting to suction again.

Suctioning should not be continued for longer than ten to fifteen seconds, since the client's oxygenation is compromised during this time (D). (A, B, and C) may be performed after the client is re-oxygenated and additional suctioning is performed. Correct Answer: D

The nurse is caring for a client who is receiving 24-hour total parenteral nutrition (TPN) via a central line at 54 ml/hr. When initially assessing the client, the nurse notes that the TPN solution has run out and the next TPN solution is not available. What immediate action should the nurse take? A. Infuse normal saline at a keep vein open rate. B. Discontinue the IV and flush the port with heparin. C. Infuse 10 percent dextrose and water at 54 ml/hr. D. Obtain a stat blood glucose level and notify the healthcare provider.

TPN is discontinued gradually to allow the client to adjust to decreased levels of glucose. Administering 10% dextrose in water at the prescribed rate (C) will keep the client from experiencing hypoglycemia until the next TPN solution is available. The client could experience a hypoglycemic reaction if the current level of glucose (A) is not maintained or if the TPN is discontinued abruptly (B). There is no reason to obtain a stat blood glucose level (D) and the healthcare provider cannot do anything about this situation. Correct Answer: C

A client is admitted to the hospital with intractable pain. What instruction should the nurse provide the UAP who is assisting with a bed bath?

Take measures to promote as much comfort as possible

A client is admitted to the hospital with intractable pain. What instruction should the nurse provide the unlicensed assistive personnel (UAP) who is preparing to assist this client with a bed bath?

Take measures to promote as much comfort as possible.

A client is admitted to the hospital with intractable pain. What instruction should the nurse provide the unlicensed assistive personnel (UAP) who is preparing to assist this client with a bed bath? Take measures to promote as much comfort as possible. Report any signs of drug addiction to the nurse immediately. Wait until the client's pain is gone before assisting with personal care. This client's pain will be difficult to manage, since the cause is unknown.

Take measures to promote as much comfort as possible. Rationale Intractable pain is highly resistant to pain relief measures, so it is important to promote comfort (A) during all activities. A client with intractable pain may develop drug tolerance and dependence, but (B) is inappropriate for a UAP. Since intractable pain is resistant to relief measures, (C) may not be possible. Psychogenic pain (D) is a painful sensation that is perceived but has no known cause.

A Sub-Saharan African widowed immigrant woman lives with her deceased husband's brother and his family, which includes the brother-in-law's children and the widow's adult children. Each family member speaks fluent English. Surgery is recommended for this client. What is the best plan to obtain consent for surgery for this client? Obtain an interpreter to explain the procedure to the client. Encourage the client to make her own decision regarding surgery. Ask the family members to provide a clarification of the surgeon's explanation to the client. Tell the surgeon that the brother-in-law will decide after explanation of the proposed surgery is provided to him and the widow.

Tell the surgeon that the brother-in-law will decide after explanation of the proposed surgery is provided to him and the widow.

The nurse is administering medications through a nasogastric tube (NGT) which is connected to suction. After ensuring correct tube placement, what action should the nurse take next? A. Clamp the tube for 20 minutes. B. Flush the tube with water. C. Administer the medications as prescribed. D. Crush the tablets and dissolve in sterile water.

The NGT should be flushed before, after and in between each medication administered (B). Once all medications are administered, the NGT should be clamped for 20 minutes (A). (C and D) may be implemented only after the tubing has been flushed. Correct Answer: B

The nurse prepares a 1,000 ml IV of 5% dextrose and water to be infused over 8 hours. The infusion set delivers 10 drops per milliliter. The nurse should regulate the IV to administer approximately how many drops per minute? A. 80 B. 8 C. 21 D. 25

The accepted formula for figuring drops per minute is: amount to be infused in one hour × drop factor/time for infusion (min)= drops per minute. Using this formula: 1,000/8 hours = 125 ml/ hour 125 × 10 (drip factor) = 1,250 drops in one hour. 1,250/ 60 (number of minutes in one hour) = 20.8 or 21 gtt/min (C). Correct Answer: C

When assisting an 82-year-old client to ambulate, it is important for the nurse to realize that the center of gravity for an elderly person is the A. Arms. B. Upper torso. C. Head. D. Feet.

The center of gravity for adults is the hips. However, as the person grows older, a stooped posture is common because of the changes from osteoporosis and normal bone degeneration, and the knees, hips, and elbows flex. This stooped posture results in the upper torso (B) becoming the center of gravity for older persons. Although (A) is a part, or an extension of the upper torso, this is not the best and most complete answer. Correct Answer: B

When assessing a client with an indwelling urinary catheter, which observation requires the most immediate intervention by the nurse.

The clamp on the urinary drainage bag is open

When assessing a client with an indwelling urinary catheter, which observation requires the most immediate intervention by the nurse?

The clamp on the urinary drainage bag is open.

When assessing a client with an indwelling urinary catheter, which observation requires the most immediate intervention by the nurse? The drainage tubing is secured over the siderail. The clamp on the urinary drainage bag is open. There are no dependent loops in the drainage tubing. The urinary drainage bag is attached to the bed frame.

The clamp on the urinary drainage bag is open.

A male client tells the nurse that he does not know where he is or what year it is. What data should the nurse document that is most accurate? A. demonstrates loss of remote memory. B. exhibits expressive dysphasia. C. has a diminished attention span. D. is disoriented to place and time.

The client is exhibiting disorientation (D). (A) refers to memory of the distant past. The client is able to express himself without difficulty (B), and does not demonstrate a diminished attention span (C). Correct Answer: D

The nurse is teaching a client proper use of an inhaler. When should the client administer the inhaler-delivered medication to demonstrate correct use of the inhaler? A. Immediately after exhalation. B. During the inhalation. C. At the end of three inhalations. D. Immediately after inhalation.

The client should be instructed to deliver the medication during the last part of inhalation (B). After the medication is delivered, the client should remove the mouthpiece, keeping his/her lips closed and breath held for several seconds to allow for distribution of the medication. The client should not deliver the dose as stated in (A or D), and should deliver no more than two inhalations at a time (C). Correct Answer: B

An elderly male client who is unresponsive following a cerebral vascular accident (CVA) is receiving bolus enteral feedings though a gastrostomy tube. What is the best client position for administration of the bolus tube feedings? A. Prone. B. Fowler's. C. Sims'. D. Supine.

The client should be positioned in a semi-sitting (Fowler's) (B) position during feeding to decrease the occurrence of aspiration. A gastrostomy tube, known as a PEG tube, due to placement by a percutaneous endoscopic gastrostomy procedure, is inserted directly into the stomach through an incision in the abdomen for long-term administration of nutrition and hydration in the debilitated client. In (A and/or C), the client is placed on the abdomen, an unsafe position for feeding. Placing the client in (D) increases the risk of aspiration. Correct Answer: B

An elderly male client who suffered a cerebral vascular accident is receiving tube feedings via a gastrostomy tube. The nurse knows that the best position for this client during administration of the feedings is A. prone. B. Fowler's. C. Sims'. D. supine.

The client should be positioned in a semi-sitting or Fowler's (B) position during feeding, in order to decrease the chance of aspiration. A gastrostomy tube, often referred to as a PEG tube, is inserted directly into the stomach through an incision in the abdomen and is used when long-term tube feedings are needed. In (A and/or C) positions, the client would be lying on his abdomen and on the tubing. In (D), the client would be lying flat on his back which would increase the chance of aspiration. Correct Answer: B

The nurse is teaching a client with numerous allergies how to avoid allergens. Which instruction should be included in this teaching plan? A. Avoid any types of sprays, powders, and perfumes. B. Wearing a mask while cleaning will not help to avoid allergens. C. Purchase any type of clothing, but be sure it is washed before wearing it. D. Pollen count is related to hay fever, not to allergens.

The client with allergies should be instructed to reduce any exposure to pollen, dust, fumes, odors, sprays, powders, and perfumes (A). The client should be encouraged to wear a mask when working around dust or pollen (B). Clients with allergies should avoid any clothing that causes itching; washing clothes will not prevent an allergic reaction to some fabrics (C). Pollen count is related to allergens (D), and the client should be instructed to stay indoors when the pollen count is high. Correct Answer: A

Which snack food is best for the nurse to provide a client with myasthenia gravis who is at risk for altered nutritional status? A. Chocolate pudding. B. Graham crackers. C. Sugar free gelatin. D. Apple slices.

The client with myasthenia gravis is at high risk for altered nutrition because of fatigue and muscle weakness resulting in dysphagia. Snacks that are semisolid, such as pudding (A) are easy to swallow and require minimal chewing effort, and provide calories and protein. (C) does not provide any nutritional value. (B and D) require energy to chew and are more difficult to swallow than pudding. Correct Answer: A

A 73-year-old female client had a hemiarthroplasty of the left hip yesterday due to a fracture resulting from a fall. In reviewing hip precautions with the client, which instruction should the nurse include in this client's teaching plan? A. In 8 weeks you will be able to bend at the waist to reach items on the floor. B. Place a pillow between your knees while lying in bed to prevent hip dislocation. C. It is safe to use a walker to get out of bed, but you need assistance when walking. D. Take pain medication 30 minutes after your physical therapy sessions.

The client's affected hip joint following a hemiarthroplasty (partial hip replacement) is at risk of dislocation for 6 months to a year following the procedure. Hip precautions to prevent dislocation include placing a pillow between the knees to maintain abduction of the hips (B). Clients should be instructed to avoid bending at the waist (A), to seek assistance for both standing and walking until they are stable on a walker or cane (C), and to take pain medication 20 to 30 minutes prior to physical therapy sessions, rather than waiting until the pain level is high after their therapy. Correct Answer: B

The nurse observes that a male client has removed the covering from an ice pack applied to his knee. What action should the nurse take first? A. Observe the appearance of the skin under the ice pack. B. Instruct the client regarding the need for the covering. C. Reapply the covering after filling with fresh ice. D. Ask the client how long the ice was applied to the skin.

The first action taken by the nurse should be to assess the skin for any possible thermal injury (A). If no injury to the skin has occurred, the nurse can take the other actions (B, C, and D) as needed. Correct Answer: A

An elderly client who requires frequent monitoring fell and fractured a hip. Which nurse is at greatest risk for a malpractice judgment? A. A nurse who worked the 7 to 3 shift at the hospital and wrote poor nursing notes. B. The nurse assigned to care for the client who was at lunch at the time of the fall. C. The nurse who transferred the client to the chair when the fall occurred. D. The charge nurse who completed rounds 30 minutes before the fall occurred.

The four elements of malpractice are: breach of duty owed, failure to adhere to the recognized standard of care, direct causation of injury, and evidence of actual injury. The hip fracture is the actual injury and the standard of care was "frequent monitoring." (C) implies that duty was owed and the injury occurred while the nurse was in charge of the client's care. There is no evidence of negligence in (A, B, and D). Correct Answer: C

The nurse is completing a mental assessment for a client who is demonstrating slow thought processes, personality changes, and emotional lability. Which area of the brain controls these neuro-cognitive functions? A. Thalamus. B. Hypothalamus. C. Frontal lobe. D. Parietal lobe.

The frontal lobe (C) of the cerebrum controls higher mental activities, such as memory, intellect, language, emotions, and personality. (A) is an afferent relay center in the brain that directs impulses to the cerebral cortex. (B) regulates body temperature, appetite, maintains a wakeful state, and links higher centers with the autonomic nervous and endocrine systems, such as the pituitary. (D) is the location of sensory and motor functions. Correct Answer: C

During a visit to the outpatient clinic, the nurse assesses a client with severe osteoarthritis using a goniometer. Which finding should the nurse expect to measure? A. Adequate venous blood flow to the lower extremities. B. Estimated amount of body fat by an underarm skinfold. C. Degree of flexion and extension of the client's knee joint. D. Change in the circumference of the joint in centimeters.

The goniometer is a two-piece ruler that is jointed in the middle with a protractor-type measuring device that is placed over a joint as the individual extends or flexes the joint to measure the degrees of flexion and extension on the protractor (C). A doppler is used to measure blood flow (A). Calipers are used to measure body fat (B). A tape measure is used to measure circumference of body parts (D). Correct Answer: C

The nurse notes that there are numerous scatter rugs throughout the house.

The home health nurse visits an elderly female client who had a stroke three months ago and is now able to ambulate with the assistance of a quad cane. Which assessment finding has the greatest implications for this client's care?

A female client with a nasogastric tube attached to low suction states that she is nauseated. The nurse assesses that there has been no drainage through the nasogastric tube in the last two hours. What action should the nurse take first? A. Irrigate the nasogastric tube with sterile normal saline. B. Reposition the client on her side. C. Advance the nasogastric tube an additional five centimeters. D. Administer an intravenous antiemetic prescribed for PRN use.

The immediate priority is to determine if the tube is functioning correctly, which would then relieve the client's nausea. The least invasive intervention, (B), should be attempted first, followed by (A and C), unless either of these interventions is contraindicated. If these measures are unsuccessful, the client may require an antiemetic (D). Correct Answer: B

A client is to receive cimetidine (Tagamet) 300 mg q6h IVPB. The preparation arrives from the pharmacy diluted in 50 ml of 0.9% NaCl. The nurse plans to administer the IVPB dose over 20 minutes. For how many ml/hr should the infusion pump be set to deliver the secondary infusion?

The infusion rate is calculated as a ratio proportion problem, i.e., 50 ml/ 20 min : x ml/ 60 min. Multiply extremes and means 50 × 60 /20x 1= 300/20=150 Correct Answer: 150

An unlicensed assistive personnel (UAP) places a client in a left lateral position prior to administering a soap suds enema. Which instruction should the nurse provide the UAP? A. Position the client on the right side of the bed in reverse Trendelenburg. B. Fill the enema container with 1000 ml of warm water and 5 ml of castile soap. C. Reposition in a Sim's position with the client's weight on the anterior ilium. D. Raise the side rails on both sides of the bed and elevate the bed to waist level.

The left sided Sims' position allows the enema solution to follow the anatomical course of the intestines and allows the best overall results, so the UAP should reposition the client in the Sims' position, which distributes the client's weight to the anterior ilium (C). (A) is inaccurate. (B and D) should be implemented once the client is positioned. Correct Answer: C

A client who is in hospice care complains of increasing amounts of pain. The healthcare provider prescribes an analgesic every four hours as needed. Which action should the nurse implement? A. Give an around-the-clock schedule for administration of analgesics. B. Administer analgesic medication as needed when the pain is severe. C. Provide medication to keep the client sedated and unaware of stimuli. D. Offer a medication-free period so that the client can do daily activities.

The most effective management of pain is achieved using an around-the-clock schedule that provides analgesic medications on a regular basis (A) and in a timely manner. Analgesics are less effective if pain persists until it is severe, so an analgesic medication should be administered before the client's pain peaks (B). Providing comfort is a priority for the client who is dying, but sedation that impairs the client's ability to interact and experience the time before life ends should be minimized (C). Offering a medication-free period allows the serum drug level to fall, which is not an effective method to manage chronic pain (D). Correct Answer: A

The nurse observes an unlicensed assistive personnel (UAP) taking a client's blood pressure with a cuff that is too small, but the blood pressure reading obtained is within the client's usual range. What action is most important for the nurse to implement? A. Tell the UAP to use a larger cuff at the next scheduled assessment. B. Reassess the client's blood pressure using a larger cuff. C. Have the unit educator review this procedure with the UAPs. D. Teach the UAP the correct technique for assessing blood pressure.

The most important action is to ensure that an accurate BP reading is obtained. The nurse should reassess the BP with the correct size cuff (B). Reassessment should not be postponed (A). Though (C and D) are likely indicated, these actions do not have the priority of (B). Correct Answer: B

6.Which technique is most important for the nurse to implement when performing a physical assessment? A. A head-to-toe approach. B. The medical systems model. C. A consistent, systematic approach. D. An approach related to a nursing model.

The most important factor in performing a physical assessment is following a consistent and systematic technique (C) each time an assessment is performed to minimize variation in sequence which may increase the likelihood of omitting a step or exam of an isolated area. The method of completing a physical assessment (A, B, and D) may be at the discretion of the examiner, but a consistent sequence by the examiner provides a reliable method to ensure thorough review of the clients' history, complaints, or body systems. Correct Answer: C

An obese male client discusses with the nurse his plans to begin a long-term weight loss regimen. In addition to dietary changes, he plans to begin an intensive aerobic exercise program 3 to 4 times a week and to take stress management classes. After praising the client for his decision, which instruction is most important for the nurse to provide? A. Be sure to have a complete physical examination before beginning your planned exercise program. B. Be careful that the exercise program doesn't simply add to your stress level, making you want to eat more. C. Increased exercise helps to reduce stress, so you may not need to spend money on a stress management class. D. Make sure to monitor your weight loss regularly to provide a sense of accomplishment and motivation.

The most important teaching is (A), so that the client will not begin a dangerous level of exercise when he is not sufficiently fit. This might result in chest pain, a heart attack, or stroke. (B, C, and D) are important instructions, but are of less priority than (A). Correct Answer: A

Turn the client q2h.

The nurse assesses an immobile, elderly male client and determines that his blood pressure is 138/60, his temperature is 95.8 F, and his output is 100 ml of concentrated urine during the last hour. He has wet-sounding lung sounds, and increased respiratory secretions. Based on these assessment findings, what nursing action is most important for the nurse to implement?

Active ROM exercises to both arms and legs two or three times a day.

The nurse is caring for a client who is weak from inactivity because of a 2-week hospitalization. In planning care for the client, the nurse should include which range of motion (ROM) exercises?

Pulse rate decreases from 78 to 52 beats/min.

The nurse is digitally removing a fecal impaction for a client. The nurse should stop the procedure and take corrective action if which client reaction is noted?

Vitamin B12.

The nurse is discussing dietary preferences with a client who adheres to a vegan diet. Which dietary supplement should the nurse encourage the client to include in the dietary plan?

Pre-medicate the client with an analgesic, Inform the client of the plan for moving to the chair, Ask the client to push the IV pole to the chair, Assess the client's blood pressure.

The nurse is preparing a male client who has an indwelling catheter and an IV infusion to ambulate from the bed to a chair for the first time following abdominal surgery. What action(s) should the nurse implement prior to assisting the client to the chair? (Select all that apply.)

Cassette infusion pump.

The nurse is preparing to give a client dehydration IV fluids delivered at a continuous rate of 175 ml/hour. Which infusion device should the nurse use?

Draw up the irrigating solution into the syringe.

The nurse is preparing to irrigate a client's indwelling urinary catheter using an open technique. What action should the nurse take after applying gloves?

Cradle the client's heel.

The nurse is providing passive range of motion (ROM) exercises to the hip and knee for a client who is unconscious. After supporting the client's knee with one hand, what action should the nurse take next?

The home health nurse visits an elderly female client who had a stroke three months ago and is now able to ambulate with the assistance of a quad cane. Which assessment finding has the greatest implications for this client's care?

The nurse notes that there are numerous scatter rugs throughout the house.

The home health nurse visits an elderly female client who had a stroke three months ago and is now able to ambulate with the assistance of a quad cane. Which assessment finding has the greatest implications for this client's care? The husband, who is the caregiver, begins to weep when the nurse asks how he is doing. The client tells the nurse that she does not have much of an appetite today. The nurse notes that there are numerous scatter rugs throughout the house. The client's pulse rate is 10 beats higher than it was at the last visit one week ago.

The nurse notes that there are numerous scatter rugs throughout the house.

The home health nurse visits an elderly female client who had a brain attack 3 months ago & is now able to ambulate with the assistance of a quad cane. Which assessment finding has the greatest implications for this client's care?

The nurse notes there are numerous scatter rugs throughout the house

One-inch pressure sore draining serous fluid.

The nurse removes the dressing on a client's heel that is covering a pressure sore one-inch in diameter and finds that there is straw-colored drainage seeping from the wound. What description of this finding should the nurse include in the client's record?

An elderly resident of a long-term care facility is no longer able to perform self-care and is becoming progressively weaker. The resident previously requested that no resuscitative efforts be performed, and the family requests hospice care. What action should the nurse implement first? A. Reaffirm the client's desire for no resuscitative efforts. B. Transfer the client to a hospice inpatient facility. C. Prepare the family for the client's impending death. D. Notify the healthcare provider of the family's request.

The nurse should first communicate with the healthcare provider (D). Hospice care is provided for clients with a limited life expectancy, which must be identified by the healthcare provider. (A) is not necessary at this time. Once the healthcare provider supports the transfer to hospice care, the nurse can collaborate with the hospice staff and healthcare provider to determine when (B and C) should be implemented. Correct Answer: D

59.An older female client with rheumatoid arthritis is complaining of severe joint pain that is caused by the weight of the linen on her legs. What action should the nurse implement first? A. Apply flannel pajamas to provide warmth. B. Administer a PRN dose of ibuprofen. C. Perform range of motion exercises in a warm tub. D. Drape the sheets over the footboard of the bed.

The nurse should first provide an immediate comfort measure to address the client's complaint about the linens and drape the linens over the footboard of the bed (D) instead of tucking them under the mattress, which can add pressure perceived by the client as the source of her pain. (A, B, and C) may be components of the client's plan of care, but the nurse should first address the client's complaint. Correct Answer: D

The nurse witnesses the signature of a client who has signed an informed consent. Which statement best explains this nursing responsibility? A. The client voluntarily signed the form. B. The client fully understands the procedure. C. The client agrees with the procedure to be done. D. The client authorizes continued treatment.

The nurse signs the consent form to witness that the client voluntarily signs the consent (A), that the client's signature is authentic, and that the client is otherwise competent to give consent. It is the healthcare provider's responsibility to ensure the client fully understands the procedure (B). The nurse's signature does not indicate (C or D). Correct Answer: A

An older client who requires frequent monitoring fell and fractured a hip. Which nurse is at greatest risk for a malpractice judgment? The nurse who worked the 7 to 3 shift at the hospital and wrote poor nursing notes. The nurse assigned to care for the client who was at lunch at the time of the fall. The nurse who transferred the client to the chair when the fall occurred. The charge nurse who completed rounds 30 minutes before the fall occurred.

The nurse who transferred the client to the chair when the fall occurred.

When assessing a client with wrist restraints, the nurse observes that the fingers on the right hand are blue. What action should the nurse implement first? A. Loosen the right wrist restraint. B. Apply a pulse oximeter to the right hand. C. Compare hand color bilaterally. D. Palpate the right radial pulse.

The priority nursing action is to restore circulation by loosening the restraint (A), because blue fingers (cyanosis) indicates decreased circulation. (C and D) are also important nursing interventions, but do not have the priority of (A). Pulse oximetry (B) measures the saturation of hemoglobin with oxygen and is not indicated in situations where the cyanosis is related to mechanical compression (the restraints). Correct Answer: A

An adult male client with a history of hypertension tells the nurse that he is tired of taking antihypertensive medications and is going to try spiritual meditation instead. What should be the nurse's first response? A. It is important that you continue your medication while learning to meditate. B. Spiritual meditation requires a time commitment of 15 to 20 minutes daily. C. Obtain your healthcare provider's permission before starting meditation. D. Complementary therapy and western medicine can be effective for you.

The prolonged practice of meditation may lead to a reduced need for antihypertensive medications. However, the medications must be continued (A) while the physiologic response to meditation is monitored. (B) is not as important as continuing the medication. The healthcare provider should be informed, but permission is not required to meditate (C). Although it is true that this complimentary therapy might be effective (D), it is essential that the client continue with antihypertensive medications until the effect of meditation can be measured. Correct Answer: A

While instructing a male client's wife in the performance of passive range-of-motion exercises to his contracted shoulder, the nurse observes that she is holding his arm above and below the elbow. What nursing action should the nurse implement? A. Acknowledge that she is supporting the arm correctly. B. Encourage her to keep the joint covered to maintain warmth. C. Reinforce the need to grip directly under the joint for better support. D. Instruct her to grip directly over the joint for better motion.

The wife is performing the passive ROM correctly, therefore the nurse should acknowledge this fact (A). The joint that is being exercised should be uncovered (B) while the rest of the body should remain covered for warmth and privacy. (C and D) do not provide adequate support to the joint while still allowing for joint movement. Correct Answer: A

A male client being discharged with a prescription for the bronchodilator theophylline tells the nurse that he understands he is to take three doses of the medication each day. Since, at the time of discharge, timed-release capsules are not available, which dosing schedule should the nurse advise the client to follow? A. 9 a.m., 1 p.m., and 5 p.m. B. 8 a.m., 4 p.m., and midnight. C. Before breakfast, before lunch and before dinner. D. With breakfast, with lunch, and with dinner.

Theophylline should be administered on a regular around-the-clock schedule (B) to provide the best bronchodilating effect and reduce the potential for adverse effects. (A, C, and D) do not provide around-the-clock dosing. Food may alter absorption of the medication (D). Correct Answer: B

An elderly client with a fractured left hip is on strict bedrest. Which nursing measure is essential to the client's nursing care? A. Massage any reddened areas for at least five minutes. B. Encourage active range of motion exercises on extremities. C. Position the client laterally, prone, and dorsally in sequence. D. Gently lift the client when moving into a desired position.

To avoid shearing forces when repositioning, the client should be lifted gently across a surface (D). Reddened areas should not be massaged (A) since this may increase the damage to already traumatized skin. To control pain and muscle spasms, active range of motion (B) may be limited on the affected leg. The position described in (C) is contraindicated for a client with a fractured left hip. Correct Answer: D

A client is in the radiology department at 0900 when the prescription levofloxacin (Levaquin) 500 mg IV q24h is scheduled to be administered. The client returns to the unit at 1300. What is the best intervention for the nurse to implement? A. Contact the healthcare provider and complete a medication variance form. B. Administer the Levaquin at 1300 and resume the 0900 schedule in the morning. C. Notify the charge nurse and complete an incident report to explain the missed dose. D. Give the missed dose at 1300 and change the schedule to administer daily at 1300.

To ensure that a therapeutic level of medication is maintained, the nurse should administer the missed dose as soon as possible, and revise the administration schedule accordingly to prevent dangerously increasing the level of the medication in the bloodstream (D). The nurse should document the reason for the late dose, but (A and C) are not warranted. (B) could result in increased blood levels of the drug. Correct Answer: D

The nurse assesses an immobile, elderly male client & determines that his blood pressure is 138/60, his temperature is 95.8F & his output is 100 mL of concentrated urine during the last hour. He has wet sounding lungs & increased respiratory secretions. Based on these assessment findings , what nursing action is most important for the nurse to implement?

Turn the client q2h Rationale: It will help move & drain respiratory secretions & prevent pneumonia from occurring.

The nurse assesses an immobile, elderly male client and determines that his blood pressure is 138/60, his temperature is 95.8 F, and his output is 100 ml of concentrated urine during the last hour. He has wet-sounding lung sounds, and increased respiratory secretions. Based on these assessment findings, what nursing action is most important for the nurse to implement?

Turn the client q2h.

The nurse assesses an immobile, elderly male client and determines that his blood pressure is 138/60, his temperature is 95.8 F, and his output is 100 ml of concentrated urine during the last hour. He has wet-sounding lung sounds, and increased respiratory secretions. Based on these assessment findings, what nursing action is most important for the nurse to implement? Administer a PRN antihypertensive prescription. Provide the client with an additional blanket. Encourage additional fluid intake. Turn the client q2h.

Turn the client q2h.

Which statement correctly identifies a written learning objective for a client with PVD?

Upon discharge, the client will list 3 ways to protect feet from injury

Which statement correctly identifies a written learning objective for a client with peripheral vascular disease?

Upon discharge, the client will list three ways to protect the feet from injury.

Which statement correctly identifies a written learning objective for a client with peripheral vascular disease? The nurse will provide client instruction for daily foot care. The client will demonstrate proper trimming toenail technique. Upon discharge, the client will list three ways to protect the feet from injury. After instruction, the nurse will ensure the client understands foot care rationale.

Upon discharge, the client will list three ways to protect the feet from injury.

Which nutritional assessment data should the nurse collect to best reflect total muscle mass in an adolescent? A. Height in inches or centimeters. B. Weight in kilograms or pounds. C. Triceps skin fold thickness. D. Upper arm circumference.

Upper arm circumference (D) is an indirect measure of muscle mass. (A and B) do not distinguish between fat (adipose) and muscularity. (C) is a measure of body fat. Correct Answer: D

The nurse is assisting an 82-year-old client to ambulate. Which is the center of gravity for an elderly person? Arms. Upper torso. Head. Feet.

Upper torso.

Which intervention is most important for the nurse to implement for a male client who is experiencing urinary retention? A. Apply a condom catheter. B. Apply a skin protectant. C. Encourage increased fluid intake. D. Assess for bladder distention.

Urinary retention is the inability to void all urine collected in the bladder, which leads to uncomfortable bladder distention (D). (A and B) are useful actions to protect the skin of a client with urinary incontinence. (C) may worsen the bladder distention. Correct Answer: D

A male client has a nursing diagnosis of "spiritual distress." What intervention is best for the nurse to implement when caring for this client?

Use reflective listening techniques when the client expresses spiritual doubts.

Male client with nursing diagnosis of "spiritual distress". What intervention is best for the nurse to implement when caring for this client?

Use reflective listening techniques when the client expresses spiritual doubts. Rationale: Client should be consulted before involving chaplain.

The healthcare provider prescribes morphine sulfate 4mg IM STAT. Morphine comes in 8 mg per ml. How many ml should the nurse administer? A. 0.5 ml. B. 1 ml. C. 1.5 ml. D. 2 ml.

Using ratio and proportion: 8mg: 1ml :: 4mg:Xml 8X=4 X=0.5 Correct Answer: A

What is the most important reason for starting intravenous infusions in the upper extremities rather than the lower extremities of adults? A. It is more difficult to find a superficial vein in the feet and ankles. B. A decreased flow rate could result in the formation of a thrombosis. C. A cannulated extremity is more difficult to move when the leg or foot is used. D. Veins are located deep in the feet and ankles, resulting in a more painful procedure.

Venous return is usually better in the upper extremities. Cannulation of the veins in the lower extremities increases the risk of thrombus formation (B) which, if dislodged, could be life-threatening. Superficial veins are often very easy (A) to find in the feet and legs. Handling a leg or foot with an IV (C) is probably not any more difficult than handling an arm or hand. Even if the nurse did believe moving a cannulated leg was more difficult, this is not the most important reason for using the upper extremities. Pain (D) is not a consideration. Correct Answer: B

The nurse is discussing dietary preferences with a client who adheres to a vegan diet. Which dietary supplement should the nurse encourage the client to include in the dietary plan? Fiber. Folate. Ascorbic acid. Vitamin B12.

Vitamin B12

The nurse is discussing dietary preferences with a client who adheres to a vegan diet. Which supplement should nurse encourage client to include in dietary plan?

Vitamin B12

The nurse is discussing dietary preferences with a client who adheres to a vegan diet. Which dietary supplement should the nurse encourage the client to include in the dietary plan?

Vitamin B12.

The nurse is discussing dietary preferences with a client who adheres to a vegan diet. Which dietary supplement should the nurse encourage the client to include in the dietary plan? Fiber. Folate. Ascorbic acid. Vitamin B12.

Vitamin B12. Rationale Vitamin B12 is normally found in liver, kidney, meat, fish and dairy products. A vegan who consumes only vegetables without careful dietary planning and supplementation may develop peripheral neuropathy due to a deficiency in vitamin B12 (D). (A, B, and C) are commonly adequate in vegetables and fruits.

28. Which new prescriptions and/or medication changes should the nurse anticipate next?

Vitamin K and holding the warfarin (Coumadin) dose today

A client who is 5' 5" tall and weighs 200 pounds is scheduled for surgery the next day. What question is most important for the nurse to include during the preoperative assessment? A. What is your daily calorie consumption? B. What vitamin and mineral supplements do you take? C. Do you feel that you are overweight? D. Will a clear liquid diet be okay after surgery?

Vitamin and mineral supplements (B) may impact medications used during the operative period. (A and C) are appropriate questions for long-term dietary counseling. The nature of the surgery and anesthesia will determine the need for a clear liquid diet (D), rather than the client's preference. Correct Answer: B

A client is admitted with a stage 4 pressure ulcer that has a black, hardened surface & a light pink wound bed with malodorous green drainage. Which dressing is best for the nurse to use first?

Wet to moist

Asks permission before touching a client.

What action by the nurse demonstrates culturally sensitive care?

Position prone with a small pillow below the diaphragm.

What action should the nurse implement to prevent the formation of a sacral ulcer for a client who is immobile?

Consider the sterile field contaminated if it becomes wet during the procedure.

What action should the nurse implement when adding sterile liquids to a sterile field?

Examine the effectiveness of nursing interventions toward meeting client outcomes.

What activity should the nurse use in the evaluation phase of the nursing process?

Continue the planned nursing interventions to restore the client's fluid volume.

When assessing a client with a nursing diagnosis of fluid volume deficit, the nurse notes that the client's skin over the sternum "tents" when gently pinched. Which action should the nurse implement?

Impaired gas exchange.

When caring for an immobile client, what nursing diagnosis has the highest priority?

Locate the perineum.

When teaching a female client to perform intermittent self-catheterization, the nurse should ensure the client's ability to perform which action?

To obtain the most complete assessment data for a client with chronic pain, which information should the nurse obtain?

Which activities during a routine day are impacted by your pain?

To obtain the most complete assessment data for a client with chronic pain, which information should the nurse obtain? Can you describe where your pain is the most severe? What is your pain intensity on a scale of 1 to 10? Is your pain best described as aching, throbbing, or sharp? Which activities during a routine day are impacted by your pain?

Which activities during a routine day are impacted by your pain?

To obtain the most complete assessment for a client with chronic pain, which information should the nurse obtain?

Which activities during a routine day are impacted by your pain?

Respiratory rate.

Which client assessment data is most important for the nurse to consider before ambulating a postoperative client?

Emptying the urinary catheter drainage bag for a client with Alzheimer's disease.

Which client care activity requires the nurse to wear barrier gloves as required by the protocol for Standard Precautions?

Obtain a prescription for removal of the catheter as soon as possible.

Which nursing intervention is most beneficial in reducing the risk of urosepsis in a hospitalized client with an indwelling urinary catheter?

The client signs a document that designates another person to make legally binding healthcare decisions if client is unable to do so.

Which statement best describes durable power of attorney for health care?

Upon discharge, the client will list three ways to protect the feet from injury.

Which statement correctly identifies a written learning objective for a client with peripheral vascular disease?

Ineffective coping related to response to positive biopsy test results.

Which statement is an example of a correctly written nursing diagnosis statement?

Speak initially with the oldest family member to show respect.

While caring for a child and mother from Cambodia, what action should the nurse implement to accommodate the clients' cultural needs?

A nurse is preparing to insert a rectal suppository & observes a small amount of rectal bleeding. What action should the nurse implement?

Withhold the administration of the suppository until contacting the HCP

A nurse is preparing to insert a rectal suppository and observes a small amount of rectal bleeding. What action should the nurse implement?

Withhold the administration of the suppository until contacting the healthcare provider.

A client arrives for scheduled needle aspiration. He tells the nurse he has already given verbal consent to the HCP. What action should the nurse implement?

Witness the client's signature on the consent form

A male client arrives at the outpatient surgery center for a scheduled needle aspiration of the knee. He tells the nurse that he has already given verbal consent for the procedure to the healthcare provider. What action should the nurse implement?

Witness the client's signature on the consent form.

The healthcare provider prescribes 1,000 ml of Ringer's Lactate with 30 Units of Pitocin to run in over 4 hours for a client who has just delivered a 10 pound infant by cesarean section. The tubing has been changed to a 20 gtt/ml administration set. The nurse plans to set the flow rate at how many gtt/min? A. 42 gtt/min. B. 83 gtt/min. C. 125 gtt/min. D. 250 gtt/min.

gtt/min = 20gtts/ml X 1000 ml/4hrs X 1 hr/60 min Correct Answer: B


Related study sets

4A: That little voice in your head

View Set

Chapter 18: The Cardiovascular System

View Set

Chapter 1: the world of innovative management

View Set

SOCI 3201 Practice Exam Questions

View Set

BIOLOGY - UNIT 8: HUMAN ANATOMY AND PHYSIOLOGY

View Set

Social Psychology Exam 2-Study Guide

View Set

Module 8: Pharmacology and Intravenous Therapies

View Set

Biology 1620 Final - Spring 2018 - USU - Gilbertson/Messina

View Set

TestOut Chapter 14 - Network Hardening

View Set